Você está na página 1de 56

bI. Why have law governing lawyers? A. Some rules are common sense B.

Lawyers held to a higher standard C. Originally drafted to create a noble professioncreate aristocracy D. Ethical obligations might differ from what is moral? 1. Giving indigent clients rent money while working on their case 2. Rules say this is unethical 3. Rules protect us from the appearance of misconduct E. Reasons to draw the lines 1. Where would it stop? Paying rent for one client, and this client, and that client. 2. One lawyer will not do something different from another lawyer II. Rules written by lawyers for benefit of lawyers III. Conflict of Interest A. Between lawyer and client B. Memo done in 30 hours v. 100 hours C. Truthfulness D. Billing disclosure IV. Lawyer Regulation A. In most states, it comes out of the Supreme Court, rather than the Legislature 1. Highest judges appoint committee to work on rules for that particular state B. There are courts that will overturn laws that have been passed by Legislatures purporting to regulate lawyers V. Admission to Practice A. Bar checks interesting things that are often listed in fitness to serve/character assessments B. Deanie on a Weanie C. Problem 1-2: POT 1. Should disclose pot usage on bar application. 2. If you plead the fifth, they call you in. 3. No case where a law student has been prosecuted for confessing on the bar 4. If you never use a truthful admission to deny someone admission to the bar, why are you asking? Whats the point? D. Disclosure is far more important than whatever you did for purposes of a bar application VI. Variety of ways that state, state bars, courts, clients, can come at any particular ethical problem or professional problempages 42-43 A. See websites on page 41 before going to partner and accusing him of unethical behavior B. Can also call the source 1. Board of Professional Responsibility 2. Calling the state bar 3. Call professors VII. Mental Health of ApplicantsRose Gowers Story, page 69 A. Why do you think the bar examiners found it important to ask about her mental health? 1. Horrible statistics about depression and lawyers

2. Trying to see if anything beyond depression affects Gower B. Which, if any, of the questions asked do you believe were appropriate and necessary, and why? C. What reforms would you suggest to improve the bar examiners process for evaluating character and fitness in Connecticut? 1. Speed up the process 2. Cut down on unnecessary, invasive questions VIII. Misconduct During Law School p. 71 A. In re Mustafa 1. Student-organization embezzlement case 2. He paid the money back; pretty honest and forthright, but denied admission to bar 3. Mustafa was later suspended and placed on probation due to similar misconduct during practice a. Not showing up at a hearing b. Not returning clients phone calls c. Commingling of personal and business funds 4. stealing will get your disbarred B. Problem 1-2: The Doctored Resume 1. Girl lied on her resume 2. What sanction should the law school impose on the student? Should the alleged conduct preclude Erica from admission to the bar? 3. This is character and fitnesswe are worried about if Erica would lie in the future, to clients or on court documents IX. Disciplinary Process A. Should lawyers be disciplined for conduct having nothing to do with the practice of law? B. If the conduct affects the character of a lawyeranything that demonstrates your character and fitness C. If you are willing to perjure yourself, you are probably willing to encourage your client to do so D. We worry about some abuse of power by professors (Peterssexual harassment by Dean of law school)plagiarism, grade bias X. Duty to Report p. 101 A. Rule 8.3: Lawyers who know that another lawyer has committed a violation of the Rules of Professional Conduct that raises a substantial question as to that lawyers honesty, trustworthiness, or fitness as a lawyer in other respects, shall inform the appropriate professional authority, except if the report would reveal information required to be kept in confidence. ---you can get your clients approval however -Himmel rule in IL, lawyer is actually punished for not turning in someone else Also, a lawyer assisting another lawyer in a treatment program is not required to report. B. Rule 5.1, 5.2, & 5.3: Responsibility of partners, managing partners, supervisory lawyers 1. 5.1 explains the responsibility of a partner or supervising lawyer for ensuring compliance with the ethical rules by subordinate lawyers, and explains

when a senior lawyer may be subject to discipline for the conduct of a subordinate lawyer 2. 5.2 explains when a subordinate lawyer is responsible for her own conduct, and under what circumstances she may follow orders without fear of discipline 3 5.3, using language nearly identical to that of Rule 5.1, explains the responsibilities of lawyers who supervise nonlawyer employees for ensuring that the employees comply with the rules of professional conduct, and explains when a lawyer may be subject to discipline based on the conduct of a nonlawyer employee -reasonable resolution of an arguable question -you have to have some justication for thinking it is reasonable -can a law student get in trouble for these? -officially no C. Problem 2-3: The Little Hearing 1. Probably not going to discipline Helman 2. Volume pays the bills 3. You cannot be thrown into the pit before you have watched other lawyers 4. Ask questions to Helman before beginning work for him (but post-offer) a. Talk to other new lawyers in firm to ask what their first day/month was like b. What case load will be like c. Ask to shadow Helman for a day d. Talk to the most recent departure in the firm i. Why did they leave? -options on p. 119 D. Legal protections for subordinate lawyers 1. Wieder casewhistle-blower 2. Wolas casebilling fraud; Kelly v. Hunton & Williams a. Whistle-blowing associates fired -it seems you are not disbarred but not rewarded -look for the next job first E. Problem 1-4: The Photographer 1. Senior partner billing for work never performed a. First, talk with boss of firm b. Then, discuss with fellow associates and partners, keep it internal c. If you report someone and it gets publicity, you will never get a recommendation, firms may not hire you d. What happens when two associates reported this in the past, and now they are not here anymore? e. Whole firm could be harmed if National Oil finds this out f. Demonstrate to National Oil and other clients that firm is proactive g. If you are a junior associate, and have the proof that the billable hours are wrong, what do you do, how do you do it? Do you have to report? i. Yes, you have to report it. ii. First, report to partnership

iii. If you get fired, you have the ability to be sued iv. Reality is most people probably would not report this F. You are a junior associate, been working all day or all week juggling 2 or 3 particular matters for different clients; so busy you havent written any hours down. Now comes time to write your billing sheets in 6 hour segments. Can you bill 30 second phone calls? 1. You are going to bill it 2. Read a one page letterbill it 3. Need to bill this for your survival XI. Duty to Protect Client Confidences A. Rule 1.6(a): Confidentiality of Information A lawyer shall not reveal information relating to representation of a client unless the client gives informed consent, the disclosure is impliedly authorized in order to carry out the representation, or the disclosure is permitted by paragraph (b) 1. Rule 1.6(a) also applies to disclosures by a lawyer that do not in themselves reveal protected information but could reasonably lead to the discovery of such information by a third person. 2. WI version: SCR 20:1.6 Confidentiality 3. Differences (see comments) a. WI(c)(1)reasonably likely instead of reasonably certain b. More leeway under WI rule 1.6 c. Shall instead of may i. Takes away discretionmore burdensome to reveal B. Information that must be protected as confidential 1. All information relating to the matter on which the lawyer is representing the client is confidential, except information that is generally known 2. Personal information relating to the client that the client would not want disclosed 3. Information learned from the client, and information learned from interviews, documents, photographs, observation, or other sources 4. Information acquired before the representation begins (such as during a preliminary consultation) and after the representation terminates 5. Notes or memoranda that the lawyer creates relating to the matter C. Violent client has a past history of violence. Can you tell anyone about this past history of violence? 1. He is a former client 2. Argument for nondisclosure a. Expectation for lawyer to not tell 3. Argument for disclosure a. Safety to society D. Problem 2-1: Your Dinner with Anna, Scene 1 1. So whats it like? Get any murders off today? 2. What can you say? In fact you spent the day working on a civil suit against the local police department on behalf of a client whose wrist was broken by a cop. Can you tell Anna this? What if anything can you tell her about the work that you have been doing? a. Nothing about specific clients cases

b. Your story v. clients story c. You can talk about subject matter as long as the geographic area you are talking about is sufficiently broad E. Problem 2-2: Your Dinner with Anna, Scene 2 1. Does not matter if you are talking with another lawyer; cannot share clientspecific information (not applicable if the other lawyer is in your own firm) 2. Can talk to another lawyer in order to get ethical advicethis is protected 3. Also depends on size of town or city 4. Can you say you are working on a police brutality case? Yes 5. That your client is suing the police for excessive force? Yes 6. Naming the bar in which the fight occurred? Maybestarts to get more detailed (if theres a fight every week, then thats more okay) 7. Officer name, date of fightNo 8. First name v. last nameNO 9. Also worried about where the dinner is; where these conversations are held 10. all this could be public, you were on the courtroom steps talking about it/ apart of it is an ongoing assessment F. Generally known 1. In the newspaper, in public reports, in a filed case G. If you reveal information in a form that identifies the client or the client matter either expressly or through reasonably ascertainable inferences, this is a disclosure that violates Rule 1.6. H. You are not in violation of Rule 1.6 if you reveal information from which your clients identity could not be ascertained, or if there was no reasonable prospect that the revelation would adversely affect your clients interests. I. How do you know if there is a reasonable prospect (Restatement) of harm to a clients interests? 1. Depends on whether a lawyer of reasonable caution, considering only the clients objectives, would regard use or disclosure in the circumstances as creating an unreasonable risk of adverse effect either to those objectives or to other interests of the client. J. Exceptions p. 171 1. The client gives informed consent -caveat for co-clients 2. The disclosure is impliedly authorized in order to carry out the representation 3. The disclosure is permitted by: a. A lawyer may reveal information relating to the representation of a client to the extent the lawyer reasonably believes necessary: i. To prevent reasonably certain death or substantial bodily harm; ii. To prevent the client from committing a crime or fraud that is reasonably certain to result in substantial injury to the financial interests or property of another and in furtherance of which the client has used or is using the lawyers services iii. To prevent, mitigate, or rectify substantial injury to the financial interests or property of another that is reasonably

certain crime or lawyers

to result or has resulted from the clients commission of a fraud in furtherance of which the client has used the services; iv. To secure legal advice about the lawyers compliance with these rules; v. To establish a claim or defense on behalf of the lawyer in a controversy between the lawyer and client, to establish a defense to a criminal charge or civil claim against the lawyer based on conduct in which the lawyer was involved, or to respond to allegations in any proceeding concerning the lawyers representation of the client; or vi. To comply with other law or a court order K. Revelation of past criminal conduct 1. Information about past criminal activity by clients should be kept confidentialno exception here for past criminal activity 2. Problem 3-3: The Missing Persons, Scene 1Robert Garrow case a. Tell client not to tell you where the bodies are b. What happens if you know where the bodies are? i. Do not investigate ii. Alert anyone? Reveal that you are Garrows lawyer? a. Persuade client to tell prosecutor where the bodies are c. Argue insanity; cut a deal with the prosecutor d. state health law p. 172 about finding dead bodies 3. Problem 3-4: The Missing Persons, Scene 2 a. Negotiate with client to get permission to talk to prosecutor b. What if prosecutor does not want to talk to you? i. Tough situation -option 1: would violate 1.6 (there are exceptions maybe the state health law) -option 2: violate 1.6 4. People v. BelgeBelge(garrow attorney) indicted for not reporting someone had died without receiving medical attentionNY public health statute a. Belge argues confidential privilege existed between he and client b. Belge protected from disclosure due to Fifth Amendment right against self-incrimination; indictment against Belge dismissed c. this ethic rule is totally contrary to community morales 5. People v. Belge (appeal by prosecutor) a. Affirmed; attorney-client privilege effectively shielded the defendantattorney from his actions which would otherwise have violated the Public Health Law 6. Problem 3-5: The Missing Persons, Scene 3---tell garrows hiding spot when hes after you? -1.6 reasonable or likely death exception? -Wisconsin 1.6 v. Model 1.6 -reasonable certain v. reasonable likely -shall reveal v. may reveal

-wis b means my client is going to kill/do crime -no disrection when it is your client (shall) -wis c is anyone is doing something that could result in harm (may) -wis b (shall-preventing commiting fraud) v. wis c2 (may-to prevent certain to result or already has resulted in which your services were used-Enron clause; your opinion letters were used) (not in the model rules)-is this a good thing to have more or less disrection L. The Risk of Future Injury or Death 1. Spaulding v. Zimmerman a. Lawyers decide not to tell Spaulding about aneurysm from car accident -Court says by not disclosing opened door for settlement to be set asidecourts dont usually look at settlements but because he is a minor -Zimmerman is driver of Spaulding car (suing insurance company) -Zimmerman (his friend) doesnt even know b. Lawyers decision to determine whether to reveal this information? c. Argument for clients decision i. Lawyers say money is more important than saving an employees life (insurance is going to pay them) ii. Small town, everyone knows everyone iii. Lawyers should not take that decision away from clients -who gets to decide settlements when the insurance company is paying the lawyer?/ is it really the zimmermans lawyer? d. Argument for lawyers decision i. Lawyer doesnt tell client because its not clients problem ii. Lawyer protecting client iii. Strategy for how you conduct negotiation belongs to lawyer iv. Lawyer can legitimately say the information they gather is confidential e. Who is the lawyer working for here? What if he tells the insurance company about this information? Who is the client? (usually not insurance company) 2. Age matters (minor); Judge from Spaulding says you only have to reveal the information to the court when you are about to make the settlement 3. Why does age matter here? Why does the court reopen the case because Spaulding is a minor? a. Adult is not in need of protection of the court; adult does not warrant as much protection b. Court says because defendants did not reveal this information to the court, Court reopens the case because Spaulding was a minor; if he was not a minor, case would not have been reopened -responsibilties in wi: this is a shall because of c1 likely death; could argue under b that the settlement is fraud M. Your Dinner With Anna, Scene 3 1. Would you warn Anna?

a. Yesprobably do not have to warn Anna specifically about Diet Kola; just in general b. Very hard to not tell a friend the dangers of something you think may deform their child 2. Standard is reasonably likely (WI) for bodily harm to occur in order to reveal the information 3. Any other options a. Preempt the trouble; have client disclose before marketing the product XII. Client Fraud A. Rules Pre-Enron 1. Prohibited from assisting a client from perpetuating a fraud 2. Permitted to violate client confidentiality if your client was about to commit fraud 3. If client had already committed fraud, and you were just giving them advice after the fact, there was no duty to reveal anything B. Sarbanes-Oxley Act 1. Requires lawyers to report any information about securities fraud to the highest officials of the corporation C. Rule 1.6(b)(2) and (3) now permit a lawyer to reveal information relating to the representation of a client to the extent the lawyer reasonably believes necessary: (2) to prevent the client from committing a crime or fraud that is reasonably certain to result in substantial in jury to the financial interests or property of another and in furtherance of which the client has used or is using the lawyers services; (3) to prevent, mitigate or rectify substantial injury to the financial interests or property of another that is reasonably certain to result or has resulted from the clients commission of a crime or fraud in furtherance of which the client has used the lawyers services D. Rule 1.6: permitted to reveal E. Rule 4.1: must reveal (requires) if you are assisting in the fraudulent act F. Not responsible for revealing fraudulent information not having anything to do with your representationSee Page 145 for other situations involving duty to reveal G. Rule 1.6. does not allow a lawyer who has not assisted a clients financial crime or fraud to make a disclosure to protect another person from injury H. Rule 1.2(d) 1. A lawyer shall not counsel a client to engage, or assist a client, in conduct that the lawyer knows is criminal or fraudulent I. Under Rule 1.6(b)(2), if the lawyer thinks that the clients plan was reasonably certain to cause substantial financial injury, the lawyer may disclose the clients plan to prevent that harm J. A lawyer is required to withdraw from representing a client if continued representation would result in a violation of the rules. Rule 1.16(a) K. A lawyer is allowed to withdraw from representing a client who persists in criminal or fraudulent conduct. Rule 1.16(b) L. What is fraud?

avoid

1. Deliberate deception 2. States that have adopted the Model Rules a. Conduct that is fraudulent under the substantive law or procedural law of the applicable jurisdiction and has a purpose to deceive 3. Torts (high standard) a. Intentionally made b. A misrepresentation to the other of a material fact, an intention, or of a law c. With the intention of inducing the other person to act or to refrain from action in reliance on the misrepresentation, if the other person can demonstrate d. Financial loss as a result of e. Having relied up on the misrepresentation 4. Criminal law (lower standard) a. Mail fraud, wire fraud, bank fraud, health care fraud, consumer fraud, securities fraud, bankruptcy fraud b. Intentional (knowing) c. Misrepresentation of d. Material facts in the context addressed by the statute 5. Contracts (lowest standard) a. Material misrepresentation that was significant for the transaction even if there was no intent to deceive 6. Fraud can occur in the act of speaking OR the omission from speaking 7. Rule 4.1(b) bars a lawyer from knowingly failing to disclose a nonconfidential material fact (that is, omitting such a fact) when disclosure is necessary to assisting a clients fraudulent act. 8. Noisy withdrawal a. Lawyers disaffirmance of a document 9. Problem 2-7: Reeses Leases a. Rule 1.6lawyer may reveal b. Rule 4.1lawyer requires revelation c. Do not reveal the fraud but stop representing Executive Leasing ServicesRule 1.1(b)(6) i. Still a document on file that you prepared that is being used for ongoing financial decisionsyou must reveal ii. Noisy withdrawaldisaffirm the document iii. Often times, the threat of a noisy withdrawal is enough to get your client to come clean d. Continuum: Stay------Noisy Withdrawal------Reveal to Bank e. Talk to at least somebody in the bank f. Economic reality, what would you do: i. Talk to client, withdraw ii. Find out something fraudulent is going on, have to talk about it g. What about the fact that you are going out of business? i. Avoid bad pressconduct a noisy withdrawalat a minimum 10. Reeses Leases Part 2Handout

a. Can you tell the bank? Yes b. Should you tell the bank? Probably c. Does it matter if you know the lawyer who may get screwed? Yes more likely to tell them d. Ethical duties: It should not matter that we know people; is not paying the new law firm for work substantial financial harm? i. Could we tell the other law firm? Yeslikely financial harm ii. Should we tell the other law firm? Probably nottheir duty to investigate their clients e. Maybe even more of a duty between law firms that know each other/friends M. Using a clients confidential information to protect the lawyers interests 1. If a lawyer needs to reveal confidences to protect her own interests, she must take steps to avoid the need for revelation, to limit its scope, or to limit the dissemination of the information 2. The lawyer is allowed to respond to an assertion that he has engaged in wrongdoing by revealing information necessary to defend himself before a suit commences 3. A lawyer may reveal confidences even if the allegation is made by an injured third party rather than by a client, and even if the lawyer is not the primary target of the allegation N. When may a lawyer reveal confidential information in self-defense? 1. To establish a claim against a client for unpaid fees 2. To defend against a claim of malpractice or other claim of civil liability against the lawyer 3. To defend against a disciplinary proceeding 4. To defend against a criminal charge O. Should the lawyer inform the client before revealing confidential information? 1. Yes. The lawyer should seek solutions that do not require the lawyer to make the revelation, but the lawyer may use the information even if the client does not consent P. Rule 1.6(b)(6) permits a lawyer to disclose confidential information to comply with a court order or with other law Situation Client waives confidentiality Disclosure needed to represent the client Past physical harms to people Threatened physical harms to people Threatened or continuing client fraud or other economic crime Scope of exception Disclosure permitted if client gives informed consent Disclosure permitted where it is impliedly authorized to carry out the representation No exception permits disclosure Disclosure permitted (but not required) by Rule 1.6(b)(1) prevent reasonably certain death or bodily harm (whether or not by the client) Disclosure to prevent the crime or fraud permitted by Rule 1.6(b)(2) if it will result with reasonable certainty in substantial injury to

10

Past client fraud or other economic crime

Lawyer needs to obtain ethics advice from another lawyer Lawyer needs to prove work done to collect a fee or to defend against misconduct charge Court orders disclosure

someones financial interests or property and the client used or is using the lawyers services to commit the crime or fraud. Disclosure required under Rule 4.1(b) if those conditions are met and the revelation is necessary to avoid assisting a criminal or fraudulent act Disclosure permitted by Rule 1.6(b)(3) to prevent, mitigate, or rectify substantial economic injury that has resulted or is reasonably certain to result in substantial financial injury, if the client used the lawyer services to commit the crime or fraud Permitted by Rule 1.6(b)(4) Permitted by Rule 1.6(b)(5) Permitted by Rule 1.6(b)(6)

XIV. Use or disclosure of confidential information for personal gain or to benefit another client A. Rule 1.8(b) 1. Conflict of interest: Current Clients: A lawyer shall not use information relating to representation of a client to the disadvantage of the client unless the client gives informed consent, except as permitted or required by these Rules B. Problem 2-8: An Investment Project 1. You would like to buy this parcel of land. Are you permitted to do so under Rule 1.8(b)? Does your answer change if your client has not yet decided whether to purchase the land? a. Rule says you can use the information as long as there is no disadvantage to the client; if the client had determined not to buy it, maybe you could b. If the client had not yet decided, what if you then decided to buy it? i. This creates a disadvantage to the client because it decreases their options; rule 1.8(b) violation c. Inform client/get consent C. Problem 2-9: Rat Poison 1. Options a. Exception to tell because there could be certain death or substantial bodily harm b. Keep talking to client c. Withdraw/tell judge this puts you in an untenable position d. Go to clients home and get permission to get the documents e. Go to sheriff f. Tell prosecutor

11

2. Real case: prosecutor found out there should be search warrants, so lawyer never did have to make any call XV. The Attorney-Client Privilege and the Work Product Doctrine A. Ethics law v. Evidence law 1. Chart on page 166 2. Source: Ethical duty Rule 1.6 states may vary in implementation of 1.6 a. Evidence common law evidence rule 3. Scope: Ethical duty to protect confidences: information relating to the representation of a client (obtained from any source) a. Evidence: narrower scope: confidential communication between a lawyer and a client for the purpose of obtaining legal advice 4. Method of enforcement: Ethics Professional discipline a. Evidence quash subpoena or otherwise exclude the revelation from evidence A. Rule 3.7: prohibits lawyers from acting as advocates and witnesses in the same trials 1. Exceptions a. If a lawyer has relevant information and no privilege applies b. Adversary lawyer is able to show a compelling and legitimate need B. Elements of attorney-client privilege 1. Communication 2. Privileged Persons 3. Communication in confidence a. It is the disclosure to a third person of the contents of a privileged communications that waives the privilege. 4. Communication for the purpose of seeking legal assistance C. The communication with the lawyer is privileged, but the underlying facts are not. D. Privilege for Corporations 1. Upjohn Co. v. United States a. In federal proceedings applying federal law, corporate entities could claim attorney-client privilege and the scope of the privilege should depend on the subject matter of the communication, not on who was doing the communicating. b. Not binding on state courts E. Crime-Fraud Exception 1. No privilege if a client seeks assistance with a crime or fraud a. Does not apply to past crimes or frauds 2. The clients intention to perform a criminal or fraudulent act triggers the crimefraud exception. It doesnt matter whether the client knows that the act is wrongful 3. Does not matter if the client did not reveal all the facts to the lawyer, so the lawyer does not know it is a criminal or fraudulent act. Only the clients intention matters, so the conversation is not privileged F. Death of a client 1. Swidler & Berlin v. United States a. Prevents disclosure of attorneys notes with deceased client G. Waiver

12

1. Attorney-client privilege can be expressly waived by the client. 2. It can also be waived by the lawyer if the client has authorized the waiver. 3. Waiver occurs by revealing privileged communication to a nonprivileged person H. Work Product Doctrine 1. Protects notes and other material that a lawyer prepares in anticipation of litigation from discovery in pretrial civil proceedings 2. It is only the lawyers need to use the information in litigation that creates a degree of protection 3. Thoughts, strategies, or mental impressions I. Problems 1. 3-1: a. None of this is privileged; videotape is confidentialinformation relating to the representation of a client (obtained from any source) b. Videotape itself is not a communication between lawyer and client c. Even though the investigator took the videotape is probably an agent of the lawyer, again, the videotape itself is not a communication between lawyer and client (client not even present when tape was made) d. Does not matterDumont (person being videotaped) was not even your client; everything videotaped was out in public so it was not even in confidence e. If private investigator comes back and reports, or writes up a report, these are privileged i. Edit the footage of the tapethis could be protected by the work product doctrine 2. 3-2: a. Privileged information; are not required to turn over b. Turning it over will minimize bad press, work in your favor for a plea deal c. May lose trust in employees though; stock prices will go down d. Turning this over will be less burdensome on company 3. 3-3: Best arguments to defeat the claim of attorney-client privilege a. Just confidentiality, you could probably defeat it under substantial financial injury b. But attorney-client privilege applies c. May have a chance because privilege does not protect underlying factsbut this may be the medical records that you already have d. Goal: reveal intent on part of Tractenberg e. Privilege should be waived because it is fraudulentcrime-fraud exception 4. 3-4: Waiver a. Confidentiality i. Could reveal under exception to rule against disclosing client confidences

13

a. Threatened physical harms to peopleRule 1.6(b)(1) to prevent reasonably certain death or bodily harm ii. Maybe Rule 1.6(b)(3)property interest in job b. Who do you tell? Judgment call i. Prosecutor or Hammers attorney? Both? ii. Probably Hammers attorney; let Hammers attorney call you as a witness iii. Depends on how much you know about prosecutor and state; if prosecutor knows you as a criminal defense attorney, go to prosecutor who gets to dismiss the charges and deal with this quietly, as opposed to a loss of trial iv. Defense attorney really is the one who wants the information and will argue for it the hardest c. Privilege i. Privilege still exists even though he is deadSwindleraffects estate, do not reveal privilege a. Estate caseprivilege does not disappear ii. Crime-fraud exception applies? iii. Prosecutor could not compel lawyer to testify about deceased client a. Californiaprivilege terminates shortly after clients death iv. Other concern is what are you protecting by not revealing this information? Nothing because client is deadcould estate sue lawyer in some way? XVI. Relationships Between Lawyers and Clients A. Formation of the lawyer-client relationship 1. Rule 1.1: Competence a. A lawyer shall provide competent representation to a client. Competent representation requires the legal knowledge, skill, thoroughness and preparation reasonably necessary for the representation. b. You may turn clients down c. For civil cases, you have blanket ability to turn clients down (just because you dont like them, or because you dont want the case; any reason) d. Not the case in criminal casescannot be so picky e. Having someone not pay you does not mean you have not established a lawyer-client relationship 2. Togstad v. Vesely, Otto, Miller & Keefe a. Togstad comes in and her husband was screwed by medical malpractice b. Miller, the lawyer, tells Togstad that she does not have a valid claim c. Miller never consults with his partner as he said he would d. Togstad loses her right to file suit because of the statute of limitations e. Togstad then sues Miller because the SOL has passed

14

f. Togstad has no duty to pursue the relationship g. Test of whether there is a relationship: i. Whether the client believes that there is a relationship, a lawyer-client relationship exists h. As a lawyer, express to the client in writing up front that you are not representing them and not in a relationship with them i. Duty of competence only applies in a lawyer-client relationship 3. Problem 4-1: The Chat Room a. Assuming that Swimmers right to sue in California is not yet barred, are you in any potential trouble? i. Noyou are not representing Swimmer, therefore, you are not bound by Rule 1.1Competent representation ii. Client should not think there is a lawyer-client relationship because of the waiver the client read that clearly says Participating in this chat room does not establish an attorneyclient relationship. b. Should you be at risk of liability for your comments in the chat room? If so, is that an example of the maxim that no good deed goes unpunished? i. Noyour advice particularly pertained to Swimmers questions ii. The fact that you cannot get back in touch with Swimmer seems to provide additional protection for the lawyer B. Lawyers responsibilities as agents 1. Express and implied authority from client to lawyer a. Actual authority 2. Apparent authorityclient based a. If a principal places an agent in a position that causes a third person reasonably to believe that the principal had given the agent express authority b. A lawyers statement to a third party that he is authorized to act does not constitute apparent authority c. Only the acts or statements of a client (or another principal) can justify reliance by the third party 3. Problem 4-2: The Fired Guard a. What arguments support your motion? i. Clients actions create the apparent authority, NOT lawyers words or actions; since Client was on the phone with Green (lawyer) the whole time during the settlement conference, you can assume that Client was acquiescing the whole time to what Green was saying, so there is apparent authority b. What arguments can you expect from Salernos new lawyer? i. Salerno gave her old lawyer apparent authority to only accept a settlement if it included her being reinstated by the county. ii. No authority to go beyond what was said over the phone c. What might you have done at the time of the settlement conference to avoid having this problem now?

15

i. Confirmed the settlement agreement with Salerno before leaving the settlement conference d. You are pretty sure that Salerno never authorized the settlement that Green agreed to. Is it fair to Salerno for you to try to enforce the agreement? If not, should you do it anyway? i. Yes, it is fair because based on my understanding, Green had apparent authority from Salerno. It is her own fault for failing to show up to the settlement conference, as mandated by the magistrate judge anyway. C. Lawyers duties of competence, honesty, communication, and diligence 1. Competence a. Rule 1.1: Competent representation requires the legal knowledge, skill, thoroughness and preparation reasonably necessary for the representation. b. A lawyer must inquire into and analyze the factual and legal elements of the problem c. Matter of Neal i. Definitely a lawyer-client relationship ii. Question of incompetence equaling the lack of filing iii. Automatic appeal = automatically granted, not automatically filed iv. A low fee paid is NOT a defense to incompetent lawyering d. Problem 4-3: The Washing Machine i. See what other options Kamath has other than returning the washing machine ii. Maybe advise him to take the settlement since he may be able to afford the settlement; avoid attorneys fees at trial iii. Investigate contract, consumer, and harassment law for a short period of time perhaps 2. Competence in criminal trialsStrickland v. Washington a. A convicted defendants claim that counsels assistance was so defective as to require reversal of a conviction or death sentence has two components: i. First, the defendant must show that counsels performance was deficient. This requires showing that counsel made errors so serious that counsel was not functioning as the counsel guaranteed the defendant by the Sixth Amendment. ii. Second, the defendant must show that the deficient performance prejudiced the defense. This requires showing that counsels errors were so serious as to deprive the defendant of a fair trial, a trial whose result is reliable. b. Marshall Dissenting i. This is ineffective representation 3. Is it ever okay to lie? a. Is the subject lied about either trivial or private? b. Is anyone harmed by the lie?

16

lawyer

c. Is the purpose of the lie to protect someone? d. Does the person lied to have a right to know (or a strong interest in knowing the truth)? e. If there is a reason to tell a lie, can the problem be solved without lying? f. If you tell this lie, will you need to tell other lies to cover up the first one? g. Instead of lying, just do not answer the question h. Think about what is the hardest question you might be askedquestion that you dread the most 4. SEE RULE 1.4 TABLE ON PAGES 234-235 a. Return clients phone calls, regardless b. Keep regular communication with your client 5. Rule 8.4(c): prohibits a lawyer from engaging in conduct involving dishonesty, fraud, deceit, or misrepresentation. 6. Breach of fiduciary duty, in the context of a lawyer-client relationship, is the lawyers failure to act consistently with the trust that a client reposes in a because the lawyer has special skills and knowledge 7. Problem 4-4: Lying to Clients a. Exaggerating expertise i. People in your firm that have expertise, you can probably pass it off to someone else in your firm ii. Do not explicitly say you are an expert iii. Say Our firm has a lot of experience iv. Tell client it is in their best interest to bring in a team of people to work on them a. What if client thinks this will now cost more money with more people? b. Say associates part of team cost less money, so this is saving you money v. Why lie? client will see the bill and see who works for them anyway b. Lowballing i. Understate what is a likely outcome ii. Might also look at the area of lawrepeat players, or unsophisticated clients iii. Might be a good idea if you deal with clients who might not know what is going on c. I never reviewed this document i. This is probably problematic d. Who did the work? i. If the partner actually reviews it, then thats usually fine (maybe say reviewed/prepared by, but not by) ii. Problem rises when junior associate does all the work and senior partner does nothing but takes responsibility for it or you put down senior partners name iii. Senior lawyer better be reviewing your work anyway

17

would

think the offers, do you do?

e. Covering up mistakes i. Miss a deadline, and now you are covering up ii. Just say you filed for a deadline because you thought more time was needed iii. As long as the extension does not harm the client, you can probably cover this up f. Blaming others for mistakes i. See Covering up mistakes 8. Contractual Duties a. Rule 1.4: requires that a lawyer keep a client reasonably informed, but a lawyer and client might agree that the lawyer give the client weekly reports b. Rule 1.5: requires that fees be reasonable and that a lawyer should inform a client as to the basis on which the fee will be determined 9. Rule 1.2page 241 a. If a client tells you they want to make an offer, you are required to make that offer b. Criminal casesclient gets the end decision (plead, go to trial) c. Clients get the ends, you get the means d. What if its a divorce case, and the client wants revenge? i. This would be problematic for your firm and client ii. You deal with the other sides lawyers all the time e. What if client wants you to be very respectful and have a very cordial negotiation because of their reputation in the community; you only way to deal with the other lawyer is to have extreme communicate only in writing, and be a bit of a jerk...what i. Civil caseswho gets to decide? a. In first scenario, try and convince client not to do it, be the voice of reason b. Hope your client calms down after the first meeting ii. Explain in either scenario what you are thinking and why you are choosing the decision you chose iii. When you and your client disagree, what happens? 10. Jones v. Barnes a. ISSUE: Does counsel have Constitutional duty to raise every nonfrivolous issue requested by D? b. REASONING: i. Whole point of being an experience advocate is to weed out weak arguments and press the few key issues that may

work a brief ii. Don't want to dilute your strong arguments just to throw in a few weak ones, especially when you have limited pages in and limited time during oral arguments

18

him what what he says

contrives him

iii. Role of a good advocate is to support your client's appeal to this best of your ability, he did this iv. CONCUR: Ethical obligation to argue ALL non-frivolous matters your client insists on, can consult w/ him telling you think is most likely to succeed, but in the end, goes v. But the Constitutional requirements were met here c. DISSENT: 6th right to assistance of counsel in all criminal prosecutions i. Seems like "assistance", especially when appointed, should mean that after you "counsel" your opinion, you should be taking your cues from the client w/ non-frivolous issues ii. Counsel is better than none, but when forcing a D accept counsel and accept what he says, it looks like the law against him and his "rights" are working against

iii. Right to defend is personal, remember the D bears consequences of the conviction! iv. Also deepens mistrust D's already have lawyers v. Undermines core values of the Constitution like autonomy and dignity, liberty 11. Problem 4-5: The Package Bomber a. Weigh options D. Clients with diminished capacity 1. Rule 1.14: (a): When a clients capacity to make adequately considered decisions in connection with a representation is diminished, whether because of minority, mental impairment, or for some other reason, the lawyer shall, as far as reasonably possible, maintain a normal client-lawyer relationship with the client. (b): When the lawyer reasonably believes that the client has diminished capacity, is at risk of substantial physical, financial or other harm unless action is taken and cannot adequately act in the clients own interest, the lawyer may take reasonably necessary protective action, including consulting with individuals or entities that have the ability to take action to protect the client and, in appropriate cases, seeking the appointment of a guardian ad litem, conservator or guardian. (c): Information relating to the representation of a client with diminished capacity is protected by Rule 1.6. When taking protective action pursuant to paragraph (b), the lawyer is impliedly authorized under Rule 1.6(a) to reveal information about the client, but only to the extent reasonably necessary to protect the clients interests. 2. A guardian ad litem is empowered to speak for the client (even contrary to the clients expressed wishes) in a particular legal matter 3. A conservator is given power to manage the financial affairs of the client, who thereby loses the power to buy, sell, and hold property.

19

4. A guardian has even more authority. The guardian manages the clients financial affairs and may make medical and other personal decisions for the client, who is thereafter the guardians ward. 5. Problem 4-6: Vinyl Windows a. If McCabe still cant decide how she wants to respond to the lawsuit, what should you do? i. Petition court to appoint guardian at litem ii. Possibly even a conservator to decide for McCabe if McCabe still cannot decide 6. Problem 4-8: The Foster Child a. Should you advocate for a particular placement for Grace at the upcoming hearing? If so, which placement? i. Your client is the child (anything above 5 start paying attention to your childs wishes) ii. Child wants to go with Mom iii. Better to be with Waite status quo iv. What research is available on success of certain child-foster care situations? Not really a right answer here E. Terminating a lawyer-client relationship 1. A lawyer has the right to retain any documents the client is entitled to until the client pays the lawyers bill or fee 2. Problem 4-9: The Candid Notes a. Must you send copies of the memoranda to Bryzanski? If you must send her the documents, should you delete your paralegals impressions? i. Yes but you can redact them (you should delete the impressions) 3. Grounds for termination before the work is completed a. When the client fires the lawyer i. A lawyer must withdraw if the client fires the lawyer. ii. A lawyer also must withdraw if the lawyers illness or loss of capacity would materially impair the representation b. When continued representation would involve unethical conduct c. When the lawyer wants to terminate the relationship i. The lawyer may withdraw if it is possible to do so without material adverse effect on the interests of the client. d. Matters in litigation i. If a lawyer has filed suit on behalf of a client or entered an appearance in a matter in litigation, the lawyer generally cannot withdraw from representation of the client without permission from the court that is to hear the case XVII. Conflicts of Interests A. SCR 20:1:7 Conflicts of interest current clients (WI) B. Possible remedies for conflicts between clients 1. Withdrawing or declining to represent a new client 2. Obtaining clients informed consent

20

3. Signing agreements that limit representation 4. Screening C. Categories of Conflicts 1. Concurrent a. Rules 1.7, 1.8: This is a conflict between two present obligations of a lawyer or between one present client and one prospective client. Sometimes a lawyer seeks informed consent from the affected clients to waive a concurrent conflict. In some cases, representation is prohibited even with consent 2. Successive a. Rule 1.9: This is a conflict between an obligation to a present client and an obligation to a former client. A lawyer may proceed despite a success conflict if the affected client consents 3. Imputedlaw firm a. Rule 1.10: This is a conflict between an obligation of one lawyer to a client and an obligation of an affiliated lawyer. One lawyer is infected by a conflict because one of his partners would face a conflict in taking on a particular client. The conflict could be between a client of lawyer A and a client of one of As partners (concurrent). There could be a conflict between a client of A and a past client of As partner (successive); the former client might have been represented by the same firm or the former firm. An imputed conflict may be waived by a client under the conditions stated in Rule 1.7 4. Conflicts for government lawyers a. Rule 1.11: Rule 1.11 addresses both successive and imputed conflicts of interest for lawyers who move between jobs in government and the private sector. These include successive conflicts if a lawyers new clients interests conflict with the interests of his former employer, and imputed conflicts, where work done by other lawyers in a firm conflicts with the interests of a lawyers former employer. Many such conflicts are resolved by informed consent D. Rule 1.7page 287 1. Rule 1.7a lawyer is prohibited from representing a client if one of the conflicts described in 1.7(a) exists, unless as per 1.7(b) the conflict is waivable by the client and the client gives informed consent to allow the lawyer to continue with the representation despite the conflict. a. Cannot represent both plaintiff and defendant b. Probably cannot represent two accused parties c. Cannot represent both buyer and seller E. Direct adversity 1. A conflict is said to involve direct adversity to the interests of a client if the lawyers conduct on behalf of one client requires the lawyer to act against the interest of another current client. If, in the course of representing one client, the lawyer takes action that could directly harm another client, there is direct adversity. a. Defense of one client will be harmful to the defense of another client

21

b. Represent two parties going after the same piece of real estate c. Cross-examining your own client for benefit of another client F. Material limitation 1. To evaluate whether a conflict is present, a lawyer must ask two questions a. How likely is it that a difference in interests will eventuate? b. If there likely is such a divergence, would it materially interfere with the lawyers advice to or representation of a client? 2. If no direct adversity, a conflict exists if representation of one client would be materially limited by another responsibility of the lawyer G. WI Step 2 to see if a conflict exists Notwithstanding the existence of a concurrent conflict of interest under par. (a), a lawyer may represent a client if: (1) The lawyer reasonably believes that the lawyer will be able to provide competent and diligent representation to each affected client; (2) The representation is not prohibited by law; (3) The representation does not involve the assertion of a claim by one client against another client represented by the lawyer in the same litigation or other proceeding before a tribunal; and (4) Each affected client gives informed consent, confirmed in a writing signed by the client H. How to evaluate conflicts 1. To resolve a concurrent conflict under Rule 1.7, a lawyer must a. Clearly identify the clients or clients; b. Determine whether a conflict of interest exists; c. Decide whether the representation may be undertaken despite the existence of a conflict (i.e., whether the conflict is consentable); and d. If so, consult with the clients affected under paragraph (a) and obtain their informed consent, confirmed in writing I. Nonconsentable conflicts 1. The question is whether the lawyer can reasonably conclude that he will be able to provide competent and diligent representation. 2. The question is what a reasonable lawyer would think J. Imputation of Concurrent Conflicts 1. Rule 1.10 a. While lawyers are associated in a firm, none of them shall knowingly represent a client when any one of them practicing alone would be prohibited from doing so by Rules 1.7 and 1.9, unless the prohibition is based on a personal interest of the prohibited lawyer and does not present a significant risk of materially limiting the representation of the client by the remaining lawyers in the firm... b. A disqualification prescribed by this rule may be waived by the affected client under the conditions stated in Rule 1.7 2. Rule 1.10(a) takes the position that A firm of lawyers is essentially one lawyer for purposes of the rules governing loyalty to the client 3. If one lawyer has a conflict, then so do all the other lawyers in the firm 4. Screening

22

a. Screening the conflicted lawyer from any contact with the matter that presents the problem 5. The Model Rules, however, do not permit screening as an alternative to client consent to avoid imputation of a conflict within a firm K. Problem 5-1: The Injured Passengers, Scene 1 1. Can you represent both Jill and Reema? What other information do you need to know to be able to answer this question? a. I do not see why not b. Does Jill or Reema have an interest to settle? c. Is there an insurance company here with some sort of limit? 2. Might there be a nonconsentable conflict in this case? a. No 3. Do you need to get informed consent before undertaking the joint representation? a. Yes 4. Assuming that you need to obtain informed consent, what would you need to disclose to the clients to obtain their consent? a. I am also representing client X who sustained injuries in the same cab ride you took b. You could just proceed forward once you had that consent L. Conflicts between current clients in civil litigation 1. Suing a current client a. Direct adversity b. You cannot sue a current client without consent 2. Problem 5-2: I Thought You Were My Lawyer! (use four steps in Rule 1.7) a. Your firm has signed contracts agreeing to represent Dori in the accident case and Kevin in the divorce case. What, if anything, should you advise the firm to do? Evaluate this conflict under Rule 1.7. i. Identify clients a. Dori PI client b. Kevin Divorce client ii. Whether there is a conflict or not a. Directly adverse? Significant risk? i. A significant risk exists iii. Consentable conflict? a. If the lawyer reasonably believes he will be able to provide competent and diligent representation to each affected client; representation is not prohibited by law; representation does not involve the assertion of a claim by one client against another client represented by the lawyer in the same litigation; each affected client gives informed consent; each affected client gives informed consent in writing b. Have to convince Dori to allow representation of Kevin; tell her if she did not consent, it would cost more in the long run, take more time

23

one of against the doing a good wife, that is a on alimony, able money coming do need your informed representation b. What if the law firm has two offices in different cities. If one office is handling Doris case and the other office is handling Kevins case, does the analysis change? i. No ii. Conversation may be different; analysis is not c. Assume you decide that the firm cannot represent both Kevin and Dori. Can the firm represent one of them? If so, can the firm choose which client to drop? i. Yes M. Cross-examining a current client 1. You cannot cross-examine your own client 2. Affects what is going in and what is going out N. Representation of co-plaintiffs or co-defendants in civil litigation 1. No direct adversity, but interests might conflict anyway a. One client might have a potential claim against the other b. If the clients are co-plaintiffs, they might be suing a defendant whose limited assets would make it impossible to satisfy both their claims c. If a settlement is proposed, the two plaintiffs might have different views on whether to settle. The defendant might try to trade one clients claim off against the other d. If a lawyer represents two defendants who each have some responsibility for the harm that is the subject of the suit, then each might seek to avoid liability by asserting that the other is responsible e. If the clients are plaintiffs in a lawsuit seeking injunctive relief as well as damages, they might disagree as to what the remedy should be 2. Problem 5-3: The Injured Passengers, Scene 2 a. If this is a good settlement for both, then that is okay b. Convey to clients their individual offers and that is up to them

c. Unfortunately, the conflict exists because you can share clients information within a law firm i. Set up the screening process d. What do you tell Kevin? Divorce attorney just discovered Kevins wife is a firms client i. Her financial independence, coverage of her medical expenses, would benefit Kevin ii. Explain to Kevin what happened (now representing his wife); tell him it turns out our other attorneys is representing her bus company; tell him we think he is job, if more money goes to your exgood thing for you (less dependent to pay alimony, etc.) so we think into her only helps you; but I consent to continue

24

3. Simultaneous representation in unrelated matters of clients whose interests are only economically adverse does not ordinarily constitute a conflict of interest O. Conflicts in public interest litigation 1. Problem 5-4: The Prisoners Dilemma a. Is there a conflict? Yes b. Just because it is legal aid does not make it any different than if a law firm c. What might be the conflict? i. Both sides objectives may not be met d. Can the parties consent? i. Imputed conflictmay be consentable; need to talk to OConnor attorney; OConnor attorney needs to talk to their client ii. You can probably get consent to this; need consent from both clients iii. Not the patients themselves, but guardians, doctors, nurses, etc. e. What do you need to do to protect yourself here if you want to proceed with informed consent? i. Bring in outside counsel for the purposes of determining this ii. Bring in outside counsel for both sides iii. Determining the conflict depends on the facts P. Taking inconsistent legal positions in litigation 1. Positional conflict a. A lawyer may make inconsistent arguments on a legal issue in different courts at different times without running afoul of the conflicts rules 2. A lawyer evaluating a positional conflict should consider whether the decision in one case is likely to affect the decision in the other, and whether the lawyer might be inclined to soft-pedal or otherwise alter one or another argument to avoid affecting the other case. 3. Factors to consider a. Whether the issue is before trial or appellate court b. Whether the issue is substantive or procedural c. The temporal relationship between the matters d. The practical significance of the issue to the immediate and long-run interests of the clients involved e. The clients reasonable expectations in retaining the lawyer 4. Look at what our output is (so that strategy may vary); and look at the input (what we have learned a. Tobacco litigation (input)as soon as one companys information got out there, in terms of strategy, health risks, etc., it was pretty easy to do the research and figure another company has probably behaved in the same way Q. Problem 5-5: Top Gun 1. Must the firm withdraw from representing the city against the gun manufacturers? a. No

25

b. Argument that this would be inconsistent legal positions: precedent hurting one of your other clients if you win this case c. Global sells to pharmacies who sell to people who die from overdoses d. Legitimate sale leading to MISUSE of the product here 2. Even if the firm is not required to withdraw because of the alleged conflict, should it withdraw to avoid making one of its most important clients uneasy? a. Noconvince Global to get over themselves b. Lot of bad publicity if Global switches law firms c. Public interest litigation is the one that gets screwed almost every time d. Do not withdraw here thoughquestion of your business model and what makes sense, how you are internally organized R. Conflicts in nonlitigation matters: Representation of both parties to a transaction 1. This rule, permitting representation but usually requiring consent, applies whenever a lawyer is approached by two clients seeking legal assistance with a common goal 2. If the two clients wind up in litigation against one another, does the lawyer have to withdraw altogether, or can the lawyer represent one against the other? a. In such a situation, representation is allowed only with the consent of the former client S. Conflicts in representation of organizationsREPRESENT THE CORPORATION 1. Rule 1.13(a) provides that A lawyer employed or retained by an organization represents the organization acting through its duly authorized constituents. 2. The important thing to remember is that if you represent an entity, you need to obtain information about other organizations and individuals that are related to the entity (owners of, owned by, officers and directors of, and so forth) so that if you are asked to take on a matter adverse to one of the related persons or entities, you can evaluate whether there is a conflict 3. Representation of the entity and employees a. Can a lawyer who represents a corporation provide legal services to individual employees of the organization? i. The lawyer must obtain informed consent from both the organization and the individual if interests are not aligned 4. What should a lawyer do if an officer or employee of an organization threatens to do something illegal or something that would harm the organization? a. Rule 1.13 requires the lawyer to serve the interests of the organization 5. When entity lawyers sit on board of directors a. Creates fiduciary duties b. Your fiduciary duty as the board of directors are clearly heightened 6. Problem 5-6: My Clients Subsidiary a. Must the firm withdraw from representing Hathaway, or may it continue?

26

the

b. Is continued work on her behalf contingent on obtaining Transports consent? i. Yes T. Joint representation in particular practice settings 1. Must get informed consent if you anticipate a conflict in representing two codefendants or two co-plaintiffs 2. Problem 5-7: Police Brutality, Scene 1 a. What potential conflicts should you consider in deciding whether you can represent Stone while your partner represents Morton? i. May not have enough facts to elaborate on conflicts ii. Stone and Morton could be pitted against each other b. Are these potential conflicts so serious that you must not represent both defendants even if you obtain their consent? i. If you have the same lawyer representing both, then you may be able to eliminate the prisoners dilemma 3. Problem 5-8: Police Brutality, Scene 2 a. If Stone, Morton, and the PBA all consent, can your firm accept the attractive PBA contract? i. Yes b. Assume that you conclude that you may seek waivers from your clients. How might your disclosure of the potential conflicts to the two criminal defendants be affected by your desire to obtain the PBA contract? i. You would be so interested in getting the contract, you may down-play any potential conflicts so they would consent ii. Potential conflicts a. Difficult to zealously defend each defendant c. If you ask your clients to waive the potential conflicts and they refuse, what are your options? Can you take on the PBA contract and ask PBA to get new counsel for Stone and Morton? i. Must represent your clients, cannot represent everyone ii. Say wait until the trial is done, then I will take on PBA; otherwise, until then I will advocate for Stone and Morton iii. Then the contract will most likely disappear when you crossexamine Gutman (on PBA Board) iv. Cannot boot low-paying clients for high-paying clients unless you reasonably didnt know a conflict existed 4. Problem 5-9: Police Brutality, Scene 3 a. Should you present the testimony of the officers who say that they saw Gutman going toward the bathroom with Alston? i. Yeshave to zealously defend your client ii. Do you warn in advance? a. ProbablyGutman may find outwitness tampering iii. As you go on, if you discover a conflict, you withdraw b. Is he likely to prevail? i. Conviction can be overturned if he can prove representation was affected by the conflict

27

ii. Just delay trial until everyone has moved on iii. Ultimately this was a nonconsentable conflict U. Conflicts in representing family members 1. Drawing up a separation agreement a. Varies state to state b. WIcollaborative lawno litigation; you could use a lawyer for mediation (not representing both parties) c. In WI, if things move forward, such as filing a divorce, you will not be okay; you can only draw up an agreement 2. Estate Planning a. Majority of couples do their estate planning together b. Also parents and children 3. Problem 5-10: Representing the McCarthys a. Analyze under Rule 1.7 i. Identify clients ii. Identify subjects of conflict iii. Consentable? Probably, may be difficult iv. Get it confirmed in writing b. Explain conflict to client (Hugh) c. Either way, the truth is going to come out here regardless d. If its not consentable, could you represent one or two of these clients? Or do you need to withdraw from all three? i. Withdraw from Maureen ii. If you keep Hugh and Joline, you still must tell Joline what is going on iii. Can keep Hugh OR Joline unless a lawsuit ensues, then you must withdraw from everyone iv. Argument you have to withdraw from all of them: a. Does this materially limit the representation? b. This knowledge adversely affects Jolines planning c. What information, if at all, can you convey? i. Still have a duty of confidentiality to Maureen, even as a former client ii. If you cannot tell Joline, then you must withdraw from all three clients (might be able to keep Hugh in his estate planning) XVIII. Conflicts Involving Former Clients, Government Lawyers, and Judges A. Nature of Conflicts Between Present and Former Clients 1. If a present and former clients interest conflict, a lawyer would refer to Rule 1.7 for guidance on protection of the interests of the present client 2. Protections for former clients are provided by Rule 1.9 Which rule to apply to conflicts involving present and former clients Situation Possible Interest Can this conflict be Rule Harmed waived?
Lawyer knows (or might have had access to) information from a former Breach of commitment to keep confidences of former client Yes, by informed consent of former client 1.9

28

client that could be used adversely to the former client New client wants lawyer to sue lawyers former client, whom lawyer represented for years on a variety of matters

-Possible adverse use of confidences of former client, depending on subject matter -Lawyers advocacy on behalf of new client might be compromised by relationship to former client

Yes, by informed consent of present and former clients, unless the conflict is so severe that the lawyer could not reasonably believe that he could provide competent and diligent representation to the new client

1.9

1.7

B. Duties to Former Clients 1. The primary duty that lawyers owe to former clients is to protect their confidences 2. A lawyer must decline any new matter that presents a substantial risk that the lawyer would make material adverse use of the former clients confidences, unless the former client consents 3. Under Rule 1.9(a), a lawyer may not do work on behalf of a new client if that work involves the same or a substantially related matter as a former representation and the new clients interests are materially adverse to the interests of the former client, unless the former client gives informed consent, confirmed in writing C. Distinguishing Present and Former Clients 1. Concurrent conflict a. Involves two present clients or one present and one prospective client 2. Successive conflict a. Involves one present client and one former client 3. What if a lawyer has a long-term client, but the lawyer has not done anything for that client in a year or more? a. If a lawyer has not formally concluded representation of the client, a court might still find that the client is a present one and that a conflict with another client should be evaluated as a concurrent conflict 4. Problem 6-1: Keeping in touch a. Is Almond a current client? i. NoAlmond always declined the need for additional services ii. You are providing information about the laws; not advice 5. Hiring and firing lawyers to create or eliminate conflicts a. Can you drop one client to represent another, more lucrative client adverse to the first client, in order to make the conflict successive rather than concurrent? i. No b. If one of the following conditions is satisfied, the lawyer may use the more lenient successive conflict standards to evaluate the conflict: i. The lawyer withdraws at the natural end point in the representation

29

ii. The client fires the lawyer for reasons other than the impending conflict iii. The client triggers a conflict for the lawyer by some action (for example, acquiring a company that is a defendant in a matter in which the lawyer represents the plaintiff) that was unforeseeable to the lawyer. In this case, the lawyers withdrawal makes this client a former client iv. The lawyer withdraws for some other good reasons (for example, if the client insists that the lawyer assist in committing a fraud or if the client refuses to pay the fees it owes) c. Sabotage i. When a client hires other firms for smaller matters to preclude an adversary from hiring those clients and creating a concurrent conflict for the firm 6. Former In-House Counsel a. If a lawyer worked on a matter on behalf of his employer corporation, the corporation is his former client as to that matter. However, the corporation may not be the lawyers former client as to matters that were going on during the lawyers employment if the lawyer did not work on them and received no confidences regarding those matters 7. See chart on page 353. D. Evaluating successive conflicts 1. Questions to ask a. Is it the same matter as the previous one? b. If not, is it substantially related to the previous one? i. Whether you would have had access/whether you could have learned confidential information 2. If one of the two questions is answered yes, the lawyer must ask whether the new clients interests are materially adverse to the interests of the former client 3. If the matter is the same or substantially related and there is material adversity, the lawyer may not go forward with the new representation without the former clients informed consent 4. There must be written confirmation of the consent 5. The same matter a. When is one matter the same matter as a previous one? i. A matter can be anything that is the subject of representation: a litigation, a transaction, a subject on which a client requests advice ii. The most obvious same matter is a single transaction, or lawsuit iii. It is the same matter if the new representation involves a document that the lawyer was involved in producingthe subject matter is the same 6. Substantial relationship

30

a. Is there some connection between the earlier matter and the new matter? i. Two matters might involve the same parties, lawsuit, legal issues, or the same (or overlapping) facts b. Whether the lawyer, in the course of her work in the first matter, would normally have learned information that could be used adversely to the former client in the second 7. Confidential information a. If the confidential information that a lawyer learned from the former client has become public, the lawyer is not precluded from representing the new client by the possession of that knowledge b. Difference between factual confidential information and legal expertise 8. Variations in the definition of substantial relationship a. Most courts take the position that the relationship between the two matters depends on whether there is factual information learned during the first matter that could be used adversely to the first client during the second representation. b. A few courts define substantial relationship to include matters that involve common legal issues 9. Material adversity a. Consent is required if the use of the former clients confidences might harm the former clients interests b. Otherwise you must withdraw E. Practice Issues Relating to Former Client Conflicts 1. Options for a lawyer who discovers a successive conflict a. Proceed without seeking consent b. Withdraw/do not accept the new matter c. Disclose conflict to former client and seek consent 2. Problem 6-2: The District Attorney a. Under Rule 1.9, should you withdraw from representing the state in the prosecution of Brick, or may you proceed? i. You can probably proceedyour representation of Brick occurred twelve years ago and involved nothing substantially related to the current charge of murder ii. Actualityprosecutor was disqualified for this F. Particular Applications of Rule 1.9 1. May a lawyer (on behalf of the new client) sue a former client, without the former clients consent, in a matter that is not substantially related to the previous representation? a. Yes. No consent is required unless the new matter is the same or substantially related and the new representation is materially adverse to interests of the former client 2. Representing the Competitor of a Former Client a. If the firm learned a great deal about the operation of the former client and that information could be used on behalf of a competitor to the disadvantage of that former client, the firm could have a serious conflict

31

3. Maritrans GP, Inc. v. Pepper, Hamilton & Scheetz a. Maritrans sues Pepper, Hamilton & Scheetz b. Pepper represented Maritranss competitors c. Factors to consider if injunction should be granted i. The extent to which the fiduciary (lawyer) was involved in its former clients affairs a. Peppers involvement was extensive as was their knowledge of sensitive information provided to them by Maritrans b. The danger of revelation of the confidences of a former client is so great that injunctive relief is warranted d. Substantially related and materially adverse test satisfied here i. Confidential information which is used to be materially adverse here is key e. Maritrans signs a waiver i. Court says this is invalid; extortion ii. One judge says it was a valid waiver, constituting informed consent f. Injunction granted to enjoin Pepper from representing Maritranss competitors G. Conflicts between the interests of a present client and a client who was represented by a lawyers former firm 1. Rule 1.9(a): A lawyer who has formerly represented a client in a matter shall not thereafter represent another person in the same or substantially related matter in which that persons interests are materially adverse to the interests of the former client unless the former client gives informed consent, confirmed in writing (applicable when clients are different, you are in the same firm) 2. Rule 1.9(b) : A lawyer shall not knowingly represent a person in the same or a substantially related matter in which a firm with which the lawyer formerly was associated had previously represented a client a. Whose interests are materially adverse to that person and b. About whom the lawyer had acquired information protected by Rules 1.6 and 1.9(c) that is material to the matter (whether you actually have confidential information, NOT whether you could have had access to confidential information) (applicable when you are switching firms and your client stays); unless the former client gives informed consent, confirmed in writing 3. Rule 1.9(c): A lawyer who has formerly represented a client in a matter or whose present or former firm has formerly represented a client in a matter shall not thereafter: a. Use information relating to the representation to the disadvantage of the former client except as these Rules would permit or require with respect to a client, or when the information has become generally known; or

32

b. Reveal information relating to the representation except as these Rules would permit or require with respect to a client 4. Remember: Current Client Rule 1.7; Former Clients 1.9 H. Problem 6-3: A Dysfunctional Family Business 1. Must you withdraw? a. Unless you can obtain consent from your former client, Henry, then yes, you probably should withdraw i. These matters are substantially related ii. Henrys interests are materially adverse to Josephs interests iii. As Henrys former lawyer, you probably obtained information protected by Rules 1.6 and 1.9(c) that is material to the matter b. This turns on the nature of representation c. Unlikely that informed consent will be given I. Rule 1.10: Permits screening (permits screening in WI as well) J. Successive Conflicts of Present and Former Government Lawyers 1. Rule 1.11 imposes less stringent standards regarding successive conflicts that arise from a lawyers present or past government service 2. Standard: Whether the lawyer was personally and substantially involved in a matter in the former work. 3. In the case of a former government lawyer, one must ask also whether the lawyer possesses confidential government information that could be used adversely in private practice 4. Conflicts of Former Government Lawyers in Private Practice a. Rule 1.11(a): Except as law may otherwise expressly permit, a lawyer who has formerly served as a public officer or employee of the government: i. is subject to Rule 1.9(c); and ii. shall not otherwise represent a client in connection with a matter in which the lawyer participated personally and substantially as a public officer or employee, unless the appropriate government agency gives its informed consent, confirmed in writing, to the representation b. This Rule requires no inquiry about the degree of adversity between the new clients interests and the governments interests c. Rule 1.11(e): As used in this Rule, the term matter includes: i. any judicial or other proceeding, application, request for a ruling or other determination, contract, claim, controversy, investigation, charge, accusation, arrest or other particular matter involving a specific party or parties, and ii. any other matter covered by the conflict of interest rules of the appropriate government agency d. Personal and Substantial Participation

33

former reasonable

i. To participate personally means directly, and includes the participation of a subordinate when actually directed by the government employee in the matter ii. Substantially means that the employees involvement must be of significance to the matter, or form a basis for a appearance of such significance

Bases for disqualification of former government lawyers: -Subsequent work involves a matter in which the government lawyer was personally and substantially involved while in the government (This can be cured by agency consent) Rule 1.11(a) -Subsequent work could involve use of confidential government information about a person known to the lawyer in a way that would materially disadvantage that person (this cannot be cured by consent) Rule 1.11(c) e. Screening of former government lawyers i. Rule 1.11(b): When a lawyer is disqualified from representation under paragraph (a), no lawyer in a firm with which that lawyer is associated may knowingly undertake or continue representation in such a matter unless: a. the disqualified lawyer is timely screened from any participation in the matter and is apportioned no part of the fee therefrom; and b. written notice is promptly given to the appropriate government agency to enable it to ascertain compliance with the provisions of this rule f. Confidential government information i. Rule 1.11(c): Except as law may otherwise expressly permit, a lawyer having information that the lawyer knows is confidential government information about a person acquired when the lawyer was a public officer or employee, may not represent a private client whose interests are adverse to that person in a matter in which the information could be used to the material disadvantage of that person. As used in this Rule, the term confidential government information means information that has been obtained under governmental authority and which, at the time this Rule is applied, the government is prohibited by law from disclosing to the public or has a legal privilege not to disclose and which is not otherwise available to the public. A firm with which that lawyer is associated may undertake or continue representation in the matter only if the disqualified lawyer is timely screened from any participation in the matter and is apportioned no part of the fee therefrom. g. Government lawyers are given more leeway

34

K. Problem 6-5: The Former Government Lawyer 1. Was it the same matter? a. Probably b. Both issues concern the bombing of Pan Am Flight 103 2. Personal and substantial? a. Yesworking with government about decisions about witnesses 3. Whether he should have withdrawn from this representation due to a conflict? a. He will argue Libya is entitled to the best defense possible b. Other than a Rule 1.11 violation, the representation is probably fine c. If the government wanted to have a motion to disqualify here, they could L. Conflicts of government lawyers who formerly worked in private practice 1. Rule 1.11(d): Except as law may otherwise expressly permit, a lawyer currently serving as a public officer or employee: a. is subject to Rules 1.7 and 1.9; and b. shall not: . . . participate in a matter in which the lawyer participated personally and substantially while in private practice or nongovernmental employment, unless the appropriate government agency gives its informed consent, confirmed in writing M. Conflicts involving judges, arbitrators, and mediators 1. Conflicts rules for sitting judges a. When should a judge disqualify himself from hearing a particular matter because of a conflict of interest? i. Has a personal bias toward a party or lawyer; ii. Knows about disputed facts in the case from personal experiences iii. Has an economic interest in a case; iv. Is related to a party, a lawyer, or someone with a substantial stake in the case; v. Knows that she has received campaign contributions of more than a specified amount from a party or lawyer; or vi. Has made a public statement that appears to commit her with respect to an issue in the case 2. Problem 6-6: The Judges Former Professor a. May not be able to rule objectively about a law professor you had b. This question goes more towards appearance from the actual impartiality; enough connection here that the judge should step down c. Is the plaintiffs lawyer likely to file a motion requesting that the judge recuse herself? i. Yesbut the risk is if the judge does not recuse herself, then she will be mad (strategic danger) d. If you thought that in talking to defense counsel that you were both concerned, as you should be based on these facts, that a joint motion is far more likely to succeed, then at that point, the judge may recuse herself before the motion is filed

35

3. Conflicts rules for former judges, law clerks, arbitrators, and mediators a. Rule 1.12(a): A lawyer shall not represent anyone in connection with a matter in which the lawyer participated personally and substantially as a judge or other adjudicative officer or law clerk to such a person or as an arbitrator, mediator or other third-party neutral, unless all parties to the proceeding give informed consent, confirmed in writing b. Personal and substantial participation i. Same as Rule 1.11 c. Imputation As long as the judge is timely screened from any participation in the matter and is apportioned no part of the fee therefrom; and written notice is promptly given to the parties and any appropriate tribunal d. Employment negotiation i. Rule 1.12(b): A lawyer shall not negotiate for employment with any person who is involved as a party or as lawyer for a party in a matter in which the lawyer is participating personally and substantially as a judge or other adjudicative officer or as an arbitrator, mediator or other third-party neutral. A lawyer serving as a law clerk to a judge or other adjudicative officer may negotiate for employment with a party or lawyer involved in a matter in which the clerk is participating personally and substantially, but only after the lawyer has notified the judge or other adjudicative officer N. Conflicts involving prospective clients 1. Rule 1.18a lawyer may still obtain confidential information meeting with a prospective client, which could later be used adversely against that client 2. See chart on page 400 3. Problem 6-7: The Mine Explosion a. May you agree to represent Schultz? You can bet your life that Boggle wont consent i. Nomaterially adverse information from Boggle to Schultz b. If you cant handle the matter, can your partner Oswald take on the matter? i. Yes, if you are effectively screened and do not receive part of the fee, and written notice is promptly given to Schultz c. Could you have avoided this problem? If so, how? i. Yesdo not answer any more phone calls from Boggle; hang up the phone as soon as Boggle begins to elaborate on the mine explosion XIX. Conflicts of Interest Between Lawyers and Clients A. Legal Fees

36

1. Hourly basis 2. Rule 1.5(a): Reasonable Fees: a. A lawyer shall not make an agreement for, charge, or collect an unreasonable fee or an unreasonable amount for expenses. The factors to be considered in determining the reasonableness of a fee include the following: (1) the time and labor required, the novelty and difficulty of the questions involved, and the skill requisite to perform the legal service properly; (2) the likelihood, if apparent to the client, that the acceptance of the particular employment will preclude other employment by the lawyer; (3) the fee customarily charged in the locality for similar legal services; (4) the amount involved and the results obtained; (5) the time limitations imposed by the client or by the circumstances; (6) the nature and length of the professional relationship with the client (7) the experience, reputation, and ability of the lawyer or lawyers performing the services; and (8) whether the fee is fixed or contingent 3. In evaluating whether a fee is reasonable, the courts tend to ask whether the fee is comparable to fees charged by other lawyers in similar cases, and whether the fee makes sense in light of the work performed and the results obtained 4. Brobeck, Phleger & Harrison v. Telex Corp.contract law a. Parties could not agree on a minimum fee for Brobeck b. Telex contends that the $1 million fee was so excessive as to render the contract unenforceable c. Whether a contract is fair or works an unconscionable hardship is determined with reference to the time when the contract was made and cannot be resolved by hindsight d. Although the minimum fee was clearly high, Telex received substantial value from Brobecks services. Such a contract is not unconscionable. e. Telex was a sophisticated client; they should be responsible for acquiescing to the $1 million fee i. Sixth and Seventh factors are important here ii. Other factors may contravene Brobecks argument f. When the client pushes for a contingent fee, the lawyer normally bills hourly (cleaner), it is a you bargained for this, you have to accept it 5. In the Matter of Fordham a. Allegation that Fordham charged client excessive fees in his operating a motor vehicle while under the influence charge

37

b. In considering whether a fee is clearly excessive . . . the first factor is the novelty and difficulty of the questions involved, and the skill requisite to perform the legal service properly. c. Fordhams inexperience in criminal defense work and OUI cases in particular cannot justify the extraordinarily high fee d. Clark (client) did not appear to have understood in any real sense the implications of choosing Fordham to represent Timothy (clients son). Fordham did not give Clark any estimate of the total expected fee or the number of $200 hours that would be required. This finding directly militates against the finding that Clark entered into the agreement with open eyes. e. The safe harbor formula would not be an appropriate rationale in this case because the amount of time Fordham spent to educate himself and represent Timothy was clearly excessive despite his good faith and diligence f. The test is whether the fee charged is clearly excessive, not whether the fee is accepted as valid or acquiesced in by the client g. Fordhams fee was clearly excessive; public reprimand ordered h. With even full client informed consent, the court says no, this is too much (focuses on factor 3fee customarily charged) 6. Communication about fee arrangements a. Rule 1.5(b): The scope of the representation and the basis or rate of the fee and expenses for which the client will be responsible shall be communicated to the client, preferably in writing, before or within a reasonable time after commencing the representation, except when the lawyer will charge a regularly represented client on the same basis or rate. Any changes in the basis or rate of the fee or expenses shall also be communicated to the client. b. What must be disclosed? i. The general nature of the legal services to be provided. ii. The basis or rate of the fee and expenses. c. Must the disclosure be in writing? i. No, unless the lawyer intends to charge a contingent fee d. Does the lawyer have to disclose fee and expense information before starting work on the matter? i. Nocan be before or within a reasonable time after commencing the representation e. Must the lawyer estimate the amount of time she will spend or the total fee? i. No, but it is good business practice to provide clients with a realistic assessment of the possible cost of the representation 7. Problem 7-1: An Unreasonable Fee? a. Evaluate under Rule 1.5(a) factors i. Factor 3 is important b. Additional information needed:

38

i. Comparative pricing ii. Complexity of the lease iii. Hourly rate/# of hours for this issue iv. Experience level of Colin Horlock v. What exactly was said between Ingrid and Colin initially c. Rule 1.4(a)(3)keeping client reasonably informed about the matter d. How could Colin avoid this? i. Keep client updated as case progresses ii. Let client know that attorneys fees are large here because of the large amount of hours iii. Do not bill $250/hour for something someone else can do for $30/hour iv. You can bill for learning (research), but at what point does this become excessive? v. USE GOOD COMMON SENSE ($70,000 recovery, $30,000 spent on legal fees) 8. Problem 7-2: Rising Prices a. How should you handle the rate increase? i. Clearly communicate the rate increase to your clients ii. Send a letter to all your clients explaining that you must increase your hourly rate and notifying them of the reasons and when the prospective increase will take effect iii. At the beginning of the lawyer-client relationship, to try and avoid these conversations, explain that annually, or at the start of a new year, prices may/will increase iv. Explain fee structure, give date that fees will increase 9. Regulation of hourly billing and billing for expenses a. A lawyer may not bill more time than she actually spends on a matter, except to the extent that she rounds up to minimum time periods (such as one-quarter or one-tenth of an hour) b. Lawyers may not bill pad ever i. No double billing ii. No recycled billing iii. No billing for clerical tasks a. Calling to schedule an appeal b. Cleaning the office iv. No billing clients or the firm for personal expenses or marking up expense receipts 10. Contingent Fees a. Rule 1.5(c): A fee may be contingent on the outcome of the matter for which the service is rendered, except in a matter in which a contingent fee is prohibited by paragraph (d) or other law. A contingent fee agreement shall be in a writing signed by the client and shall state the method by which the fee is to be determined, including the percentage or percentages that shall accrue to the lawyer in the event of settlement, trial or appeal;

39

litigation and other expenses to be deducted from the recovery; and whether such expenses are to be deducted before or after the contingent fee is calculated. b. The agreement must clearly notify the client of any expenses for which the client will be liable whether or not the client is the prevailing party c. Upon conclusion of a contingent fee matter, the lawyer shall provide the client with a written statement stating the outcome of the matter and, if there is a recovery, showing the remittance to the client and the method of its determination d. The rule requires a lawyer to disclose to a client how the lawyer intends to calculate the fee e. The rule does not specify a maximum percentage that may be charged, but contingent fees, like other fees, are subject to the requirement of Rule 1.5 that they be reasonable 11. Criminal and domestic relations casescontingent fees a. Rule 1.5(d): A lawyer shall not enter into an arrangement for, charge, or collect: (1) any fee in a domestic relations matter, the payment or amount of which is contingent upon the securing of a divorce or upon the amount of alimony or support, or property settlement in lieu thereof; or (2) a contingent fee for representing a defendant in a criminal case b. Another possible problem with the use of contingent fees in criminal cases is that clients facing criminal charges may be vulnerable and may accept fee agreements that overcompensate the lawyer 12. Forbidden and restricted fee and expense arrangements a. Rule 1.8(i) prohibits lawyers from acquiring an interest in litigation on behalf of a client, except for permitted liens and contingent fees 13. Financial assistance to a client a. Rule 1.8(e): A lawyer shall not provide financial assistance to a client in connection with pending or contemplated litigation, except that: (1) a lawyer may advance court costs and expenses of litigation, the repayment of which may be contingent on the outcome of the matter; and (2) a lawyer representing an indigent client may pay court costs and expenses of litigation on behalf of the client b. The rule allows lawyers to pay court costs and litigation expenses, including the cost of needed medical examinations or other costs to obtain evidence 14. Problem 7-3: An Impoverished Client a. May you pay these expenses? i. Noanalyze under Rule 1.8(e) 15. Publication rights

40

lawyer in substantial representation

a. Rule 1.8(d): Prior to the conclusion of representation of a client, a lawyer shall not make or negotiate an agreement giving the literary or media rights to a portrayal or account based part on information relating to the

b. If the case is quickly settled in a quiet plea bargain, no one will want to read the book. On the other hand, if there is a lengthy trial that makes lots of headlines in the newspaper, the book might be a bestseller. A lawyer in such a situation might be tempted to do things that would be bad for the client and good for the book. c. If after the case has concluded the pop star owes you a hefty fee, you and the client could agree to forgive part or all of the debt in exchange for transfer of literary or media rights 16. Advance payment of fees and nonrefundable retainers a. Lawyers often ask clients to pay a portion of the lawyers fee for a specified service before the service is performed. This payment is often referred to as a retainer b. The payment secures the lawyers availability but does not depend on the performance of any particular taskclassic or general retainer. The lawyer may have to forgo other obligations to maintain his availability c. Public policy disfavors nonrefundable retainers as inconsistent with a lawyers fiduciary obligations to his client B. Fee Disputes 1. A lawyer wants to ask each client to agree in advance not to sue the lawyer for malpractice, no matter what. Can the lawyer do that? Rule 1.8(h) says yes, but only if each client has independent legal representation in making that agreement. This means that in most cases, it is impractical to include a waiver of malpractice liability in a contract for legal services 2. Rule 1.8(h): A lawyer shall not: (1) make an agreement prospectively limiting the lawyers liability to a client for malpractice unless the client is independently represented in making the agreement, or (2) settle a claim or potential claim for such liability with an unrepresented client or former client unless that person is advised in writing of the desirability of seeking and is given a reasonable opportunity to seek the advice of independent legal counsel in connection therewith 3. Can a lawyer ask clients to agree in advance to arbitrate any disputes about the lawyers services? a. Yesif the client is carefully advised of the advantages and disadvantages of arbitration, the client gives informed consent, and the provision does not insulate the lawyer from liability that might otherwise be imposed by law 4. A lawyer should not:

41

a. commit any acts of harassment against a debtor or make a frivolous claim b. retain documents or unearned fees that should be turned over to a client as leverage to secure payment of fees c. make any false or misleading statements about the fee claim d. reveal information to a third party (or threaten to do so) to get a client to pay a fee 5. Rule 1.8(i) allows a lawyer to charge an otherwise permitted contingent fee or to acquire a lien authorized by law to secure the lawyers fee or expenses 6. Fee arbitration most common way to settle; should be disclosed to client up front 7. Most common thing for a client to do in a fee dispute is to counterclaim with malpractice that is when things get ugly C. Dividing fees with other firms or with nonlawyers 1. Rule 1.5(e): A division of a fee between lawyers who are not in the same firm may be made only if: (1) the division is in proportion to the services performed by each lawyer or each lawyer assumes joint responsibility for the representation (2) the client agrees to the arrangement, including the share each lawyer will receive, and the agreement is confirmed in writing; and (3) the total fee is reasonable 2. Under the rule quoted above, the referring lawyer may indeed collect a referral fee, and it could be a large part of the total fee, provided that she meets various conditions. a. First, she must take on financial and ethical responsibility for the representation as if the lawyers were associated in a partnership b. Second, the proposed share that each lawyer receives must be disclosed to and approved by the client, the fee-sharing arrangement must be confirmed in writing, and the total fee must be reasonable D. Conflicts with lawyers personal or business interests 1. Lawyers should avoid doing business with clients a. Lawyers have fiduciary obligations to clients. If a lawyer does business with or invests financially in a client, the lawyers interest in making money or gaining other advantages is likely to conflict with his ability to look out for his clients interests 2. Rule 1.8(a): A lawyer shall not enter into a business transaction with a client or knowingly acquire an ownership, possessory, security or other pecuniary interest adverse to a client unless: (1) the transaction and terms on which the lawyer acquires the interest are fair and reasonable to the client and are fully disclosed and transmitted in writing to the client in a manner that can be reasonably understood by the client;

42

(2) the client is advised in writing of the desirability of seeking and is given a reasonable opportunity to seek the advice of independent legal counsel on the transaction; and (3) the client gives informed consent, in a writing signed by the client, to the essential terms of the transaction and the lawyers role in the transaction, including whether the lawyer is representing the client in the transaction 3. Before you make a deal with a client, ask these questions: a. Are the terms fair to the client? b. Have you explained the terms to the client clearly and in writing? c. Have you advised the client in writing that she should get legal advice about the deal from a lawyer not associated with you? d. Has the client had a chance to get advice from another lawyer? e. Has the client given informed consent in writing to the terms of the deal and to the lawyers role in the deal? 4. Do not borrow money from clients; do not lend money to clients 5. Do not ever borrow a penny from clients funds 6. Do not sell anything to a client 7. Do not ask clients to invest in your business 8. Problem 7-4: Starting a Business a. What, if any, warnings or disclosures do you have to give to Lopez? i. Disclose about fee arbitration ii. Disclose possible conflicts b. Be careful when mixing roles (investor, business partner, lawyer) E. Gifts from clients 1. Rule 1.8(c): A lawyer shall not solicit any substantial gift from a client, including a testamentary gift, or prepare on behalf of a client an instrument giving the lawyer or a person related to the lawyer any substantial gift unless the lawyer or other recipient of the gift is related to the client. For purposes of this paragraph, related persons include a spouse, child, grandchild, parent, grandparent or other relative or individual with whom the lawyer or the client maintains a close, familial relationship F. Sexual relationships with clients 1. It is not as clear that there is anything wrong with a truly consensual sexual relationship between a lawyer and a client, especially if there is no harm to the quality of the legal representation. Perhaps there is always a possibility of adverse impact, if only because the romance might end and then the professional relationship would become strained 2. The rule does not prohibit lawyers from having romances with former clients G. Must inform clients if family member is representing adverse client 1. Non-imputed conflict (you could hand this over to a partner) H. Lawyers responsibilities to clients creditors 1. A lawyer is not a collection agency for all his clients creditors. The department store has no right to ask the lawyer to give it money that the client owes, even if the debt is legitimate and overdue.

43

2. Unless the creditor has a legitimate claim to the particular funds in the lawyers possession, the lawyers duty is to his client, not to the third party I. Notify client if you plan to withdraw fees from clients deposited funds XX. Lawyers Duties to Courts, Adversaries, and Others A. The morality of your particular actions are not solely based on objective law; they may differ depending on the client and situation; okay to have a moral compass B. Required investigation by lawyers filing civil cases 1. Rule 3.1: Meritorious claims and contentions: A lawyer shall not bring or defend a proceeding, or assert or controvert an issue therein, unless there is a basis in law and fact for doing so that is not frivolous, which includes a good faith argument for an extension, modification or reversal of existing law. A lawyer for the defendant in a criminal proceeding, or the respondent in a proceeding that could result in incarceration, may nevertheless so defend the proceeding as to require that every element of the case be established. C. Differences between Rule 3.1 and FRCP 11 some evidentiary support of the claim (FRCP 11) 1. Sanctions a. Violation of Rule 3.1 can result in bar disciplinary action against an attorney. A violation of FRCP 11 is punished not by the state bar but by the judge in the civil action, and it can result in nonmonetary directives or monetary sanctions against a lawyer or a party. 2. Safe harbor a. FRCP 11 has a Safe harbor provision not found in Rule 3.1 If an opposing party makes a motion complaining that a lawyer has violated FRCP 11, the lawyer may withdraw the allegedly frivolous pleading within 21 days after opposing counsels motion and suffer no sanction other than having to pay the attorneys fees that the opposing party incurred for making the motion. Although Rule 3.1 has no safe harbor provision, a bar counsel would be unlikely to file a charge against a lawyer for filing a frivolous case or defense that the lawyer withdrew pursuant to the safe harbor provision of the FRCP. D. What penalties may apply to lawyers who bring unsubstantiated suits? 1. Rule 11 penalties a. A lawyer who initiates a federal court lawsuit in good faith and later finds out (for example, through discovery) that the lawsuit is groundless may be subject to sanctions under FRCP 11. 2. Attorneys fees a. A federal statute provides that a lawyer who multiplies the proceedings in any case unreasonably and vexatiously may be required to pay the other partys attorneys fees. E. Liability for malicious prosecution 1. In some states, a defendant who has been sued on the basis of virtually no evidence may sue the plaintiff or the plaintiffs lawyer for the tort of malicious prosecution. Most courts impose higher burdens of proof for

44

winning such cases must usually prove

than for granting sanctions under FRCP 11. A plaintiff

a. that she won the previous suit in which she was a defendant b. that the prior suit was brought without probable cause c. that the prior suit was brought with malice (a motivation other than obtaining a proper adjudication of the case) and d. that the plaintiff was injured despite having won the prior suit F. Problem 8-1: Your visit from Paula Jones 1. What, if anything, must you do to corroborate Joness allegations before you may sue the president? a. Interview the person who told Jones that her position had been abolished; b. Interview others who witnessed the bodyguard approach Jones at the registration desk c. Interview any other hotel guests on the same floor as the President to verify that Jones fled the Presidents room d. Consider looking at pay stubs from Joness previous job to determine when she was transferred to the other position; compare that to when the President allegedly sexually harassed her G. Required investigation by prosecutors before charges are filed 1. Rule 3.8(a): Special responsibilities of a prosecutor The prosecutor in a criminal case shall . . . refrain from prosecuting a charge that the prosecutor knows is not supported by probable cause. H. Truth and falsity in litigation 1. Rule 3.3 (a) A lawyer shall not knowingly: (1) make a false statement of law or fact to a tribunal or fail to correct a false statement of material fact or law previously made to the tribunal by the lawyer; (2) fail to disclose to the tribunal legal authority in the controlling jurisdiction known to the lawyer to be directly adverse to the position of the client and not disclosed by opposing counsel; Nix (3) offer evidence that the lawyer knows to be false. If a lawyer, the lawyers client, or a witness called by the lawyer, has offered material evidence and the lawyer comes to known of its falsity, the lawyer shall take reasonable remedial measures, including, if necessary, disclosure to the tribunal. A lawyer may refuse to offer evidence, other than the testimony of a defendant in a criminal matter, that the lawyer reasonably believes is false (b) A lawyer who represents a client in an adjudicative proceeding and who knows that a person intends to engage, is engaging or has engaged in criminal or fraudulent conduct related to the proceeding shall take reasonable remedial measures, including, if necessary, disclosure to the tribunal (c) The duties stated in paragraphs (a) and (b) continue to the conclusion of the proceeding, and apply even if compliance requires disclosure of

45

information otherwise protected by Rule 1.6 (even if otherwise confidential) make an (d) In an ex parte proceeding, a lawyer shall inform the tribunal of all material facts known to the lawyer that will enable the tribunal to informed decision, whether or not the facts are adverse

2. Rule 8.4(c) Misconduct a. It is professional misconduct for a lawyer to . . . engage in conduct involving dishonesty, fraud, deceit or misrepresentation 3. Which truth-telling rule applies Who might lie or deceive Situation (Court, Lawyers obligation administrative hearing, or discovery) Lawyer Lawyer is considering making The lawyer must not do it. a false statement of fact or law Rules 3.3(a)(1), 8.4 to a judge Client Lawyer knows that her client Lawyer must counsel client is considering testifying and refrain from asking client falsely in court or in a questions that would elicit the deposition false testimony. Rule 3.3(a)(3) Civil client or witness in any Lawyer suspects but does not It lawyer reasonably proceeding know that planned testimony believes it is false, lawyer may be false; witness is not may refuse to offer the criminal defendant testimonyor may allow it. Rule 3.3(a)(3) Criminal defendant Lawyer suspects but does not If defendant insists on known that planned testimony testifying, the lawyer must may be false; witness is a allow it even if the lawyer criminal defendant reasonably believes it is false. Rule 3.3(a)(3) Client or witness Lawyer knows that her client Lawyers must counsel client to or other witness has testified correct the record; consider falsely during direct or crosswithdrawing; correct record if examination necessary to undo the effect of the false evidence. Rule 3.3(b) and (c), & Comment 10 Client or witness Witness has misled the court Lawyer may have duty to by making statements that are counsel client and correct the literally true but deceptive record. Rules 3.3(b), 8.4(c) Lawyer Lawyer knows of directly Lawyer must bring it to adverse controlling legal courts attention (and authority that has not been distinguish it or explain why it disclosed by opposing counsel is not authoritative). Rule 3.3(a)(2) Lawyer Lawyer knows of facts No need to disclose unless the adverse to clients interest, not proceeding is ex parte. Rule

46

requested in discovery or required to be disclosed by a court rule.

3.3(d)

professional

want to Parah?

4. Nix v. WhitesideA lawyers duties if a client intends to give false testimony a. Robinsons representation of Whiteside falls well within accepted standards of professional conduct and the range of reasonable conduct acceptable under Strickland b. Robinsons action, at most, deprived Whiteside of his contemplated perjury 5. Problem 8-2: Flight from Sudan, Scene 1 a. Should you tell Barragabi about your conversation with Al-Parah? i. Yes this will deter Barragabi from giving false testimony b. Should you file the affidavit? i. No c. Should you ask the court for permission to withdraw from representing Barragabi? i. Probably you cannot file the affidavit because you are unsure if the statements are true; nothing else you can do d. Did you make a mistake by doing your job too well? After learning that Al-Parah was in Canada, should you have said to Barragabi [your client], Ive found a number for Al-Parah in Canada. Of course I only call him if he will corroborate your story. Should I call Al-

i. Good idea, yes I. Misleading a court without lying 1. A partial truth is a statement that may literally be true but that deceives another person by omitting relevant information or twisting information in a way that distorts it 2. Perjury lying under oath is a crime. 3. Bronston standard a. Deliberately false statements by lawyers or witnesses may be prosecuted as perjury under federal and state perjury statutes, and lawyers who commit perjury can be disciplined for making false statements to courts b. Court declines to interpret the federal perjury law to prohibit intentionally misleading nonresponsive statements 4. Rule 7.1: When a lawyer communicates about himself or about his services, the statement is false or misleading if it contains a material misrepresentation of fact or law, or omits a fact necessary to make the statement considered as a whole not materially misleading 5. Problem 8-3: Flight from Sudan, Scene 2 a. Probably not; you do not want to shed anymore light on the fact that Massariah may be deported. Furthermore, this is just a half-truth, there is nothing false about Massariahs testimony; however, he did omit a material fact in this context

47

b. Required to docannot offer evidence that the lawyer knows to be false (Rule 3.3(a)(1)) c. Argument for not doing anythingthis is not really material (just an incomplete omission) d. Should you take steps to inform the government or the court that immigration officials withdrew the provisional recommendation that Massariah be given asylum and instead put Massariah into deportation proceedings? i. Yes 6. These questions tie both strategy and ethics 7. Prepare your witness to know what the most damaging question could be from the government lawyer J. Variations in state rules on candor to tribunals 1. Legal trilemma a. We want lawyers to investigate cases thoroughly b. Preserve lawyer-client confidentiality c. Do not want lawyers to become part of a system that makes decisions on the basis of false evidence K. False impressions created by lawyers during litigation 1. Problem 8-4: The Drug Testsee 8-4 from class. 2. Rules do not govern here, but so many bad decisions will catch up to you. 3. Do not mislead the court or tell half-truths to the judge. L. The attempt to give the jury a different picture through the swapping of pictures and the display of a civil-rights poster becomes a contemptible, despicable fraud that tarnishes the reputations of all involvedHow Simpson Lawyers Bamboozled a Jury M. Lawyers duties of truthfulness in preparing witnesses to testify 1. Rule 3.4: Fairness to Opposing Party and Counsel a. A lawyer shall not . . . (b) falsify evidence, counsel or assist a witness to testify falsely, or offer an inducement to a witness that is prohibited by law. 2. Problem 8-6: Refreshing Recollection a. May you ask Magill whether Martinelli made any statements to her to the effect that Superiors rates were lower than those of other companies? Yes b. May you tell her that Superior settled other cases favorably because the borrowers remembered that Martinelli said that Superiors interest rates were the lowest in the region. (Assume that you could share this information without disclosing any confidences of other clients) Yes c. May you remind her what Martinelli looks like? Yes d. Suppose you tell her that other customers of Martinelli have testified that he told them that Superiors rates were lower than those of other companies, and she then recalls that he told her the same thing. May you advise her to sound forceful and confident about this fact when she testifies? Yes

48

N. Concealment of physical evidence and documents 1. Duties of criminal defense lawyers with respect to evidence of crimes a. Rule 3.4(a): Fairness to Opposing Party and Counsel a. A lawyer shall not: . . . unlawfully obstruct another partys access to evidence or unlawfully alter, destroy or conceal a document or other material having potential evidentiary value. A lawyer shall not counsel or assist another person to do any such act. b. Just turn over evidence before anyone finds out you have it 2. State v. Olwell a. The attorney should not be a depository for criminal evidence which in itself has little, of any, material value for the purposes of aiding counsel in the preparation of the defense of his clients case b. To encourage lawyer-client confidentiality, however, the prosecutor when attempting to introduce such evidence at the trial, should take extreme precautions to make certain that the source of the evidence is not disclosed in the presence of the jury c. A lawyer was resisting a coroners subpoena 3. In re Ryder a. Lawyers cannot conceal evidence in order to protect criminal clients b. The FBI obtained a warrant to search a lawyers safe deposit box. As a result of the lawyers concealment of evidence, federal trial court imposed severe sanctions on the lawyer 4. People v. Meredith a. A defendant appealed his conviction based on an alleged violation of the rules of evidence (attorney-client privilege) 5. Morrell v. Statedocuments as opposed to evidence a. Cline had a duty to turn over the written document (kidnap plan) to the prosecutor, even without having been asked for it (Olwell) b. A defendant appealed his conviction alleging that his lawyers revelations deprived him of the effective assistance of counsel 6. Problem 8-7: A Revealing Portfolio a. Are you legally obliged to turn the file over to the attorney general or to keep it confidential? i. Probably do not have to turn the file over because nobody has requested them yet; you are not concealing the files because no one has asked for them yet; you do not have a proactive duty to turn over evidence b. May you return it to Cameron or, if you choose, keep it in your office? What will you do? i. You may be able to keep it in your office and argue that you were not keeping it to impair its integrity or its availability for use in the official proceeding ii. Keep the documents until there is an investigation

49

iii. Could give the files back to your client, say they are problematic, and when the investigation starts, they will be discoverable, do not destroy these iv. Best strategic advice: come forward 7. Rule 4.4: Respect for Rights of Third Persons (a) In representing a client, a lawyer shall not use means that have no substantial purpose other than to embarrass, delay, or burden a third person, or use methods of obtaining evidence that violate the legal rights of such a person (b) A lawyer who receives a document relating to the representation of the lawyers client and knows or reasonably should know that the document was inadvertently sent shall promptly notify the sender 8. Problem 8-8: The Break-In a. Return? Does any disciplinary rule require you to return the documents to Ron or to notify him that you have received them? i. Rons attorney would most likely already have this evidence; no disciplinary rule requires that you return the documents to Ron or to notify him that you have received them ii. Rule 4.4(b) does not apply; no inadvertence about this; argue that 4.4(b) means a mistake in sending something you did not intend to send; here, Cheryl intended to send these documents (although Ron did not) b. Negotiate? Would it violate any rules to use these documents in negotiating with Rons lawyer about custody or to push for a better financial settlement for Cheryl? i. No; use the documents to your advantage to save your client O. Concealment of documents and evidence in civil cases 1. At what point in a criminal or civil matter does the lawyers obligation not to conceal or destroy objects or documents begin? a. Criminal matters i. If a lawyer has no knowledge that a violation of law has been committed and no criminal investigation is foreseeable, a lawyer has no duty to turn evidence over to a prosecutor ii. In some states, the lawyers duty not to conceal tangible evidence takes effect as soon as the lawyer believes that an official investigation is about to be instituted. In other states, it does not begin until an investigation has actually started b. Civil matters i. Obligations in civil cases are governed by civil discovery rules as well as by professional responsibility rules. Soon after a civil case is commenced, a lawyer may have a duty under the pertinent rules of procedure to turn over some information to the opposing party, even in the absence of a discovery request

50

ii. Some state laws require the preservation of business records for specified periods of time even if no dispute is on the horizon. In general, when a lawsuit is pending or foreseeable, individuals and businesses have more stringent duties to protect and eventually to disclose relevant material iii. As in the case of tangible evidence, where no specific record preservation statute applies but a lawyer has some reason to believe that wrongdoing has occurred, state law varies as to when the duty to preserve evidence kicks in (that is, whether a lawsuit or government investigation must have begun, be imminent, or merely be foreseeable) iv. In any event, once a duty to preserve documents applies, relevant records and objects should be retained even if they could otherwise routinely be destroyed 2. A lawyers duties in responding to discovery requests a. Rule 3.4(d): Fairness to Opposing Party and Counsel i. A lawyer shall not . . . in pretrial procedure, make a frivolous discovery request or fail to make reasonably diligent efforts to comply with a legally proper discovery request by an opposing counsel 3. Problem 8-9: The Damaging Documents a. What will you do in response to the document request? i. Forward voicemail to every recording device you own so you always have it ii. Possibly call the bar hotline for advice to see if you could keep requests iii. Possibly object to discovery request (may not work) iv. Other possibility is to comply, turn over all documents (may be a wise choice since there are 120,000 pages of the documents) v. Turn over everything except the law memo that is internal b. In the real case, the firm objected to the first request as overbroad in time and scope c. The firm agreed to provide the other documents only at a mutually agreeable time d. Ultimately, the Wa. Supreme Ct. reversed, condemning the failing to turn overall relevant documents, and even sanctioned the firm (fraud) P. The Duty to Disclose Adverse Legal Authority 1. Rule 3.3(a)(2) prohibits a lawyer from knowingly failing to disclose legal authority in the controlling jurisdiction that the lawyer knows is directly adverse to her clients position, if an opponent has not already informed the judge of the adverse authority 2. Exceptions a. If the adverse law is from another jurisdiction, it need not be disclosed because only law from the controlling jurisdiction matters

51

b. Only authority that is directly adverse need be disclosed. Dicta and holdings that are applicable only by analogy do not have to be disclosed c. Persuasive authority, such as statements in treatises and law review articles, need not be disclosed 3. Can a lawyer be disciplined for overlooking a directly adverse case? a. No, because the prohibition is for knowingly failing to reveal the adverse authority. On the other hand, knowledge under the ethics rules can be inferred from circumstances Q. Disclosures in ex parte proceedings 1. Rule 3.3(d) requires that in ex parte proceedings, a lawyer must inform the tribunal of all material facts known to the lawyer that will enable the tribunal to make an informed decision, whether or not the facts are adverse 2. Ex parte communication with judges/Improper influences on judges a. A lawyer must not communicate with a judge about a pending case, orally or in writing, unless the lawyers for all parties to the case are privy to the communication. Rule 3.5(b) b. Rule 3.5 prohibits only communications related to particular proceedings, not personal communications R. Trial Publicity 1. Rule 3.6(a) and (b) Trial Publicity a. A lawyer who is participating or has participated in the investigation or litigation of a matter shall not make an extrajudicial statement that the lawyer knows or reasonably should known will be disseminated by means of public communication and will have a substantial likelihood of materially prejudicing an adjudicative proceeding in the matter 2. Problem 8-10: A Letter to the Editor a. Should you mail the letter? No b. What, if any, changes would you make in the letter before sending it, how would you change the letter? S. Impeachment of truthful witness 1. If you know a witness is telling the truth, is it your duty as defense attorney to try and paint her as a liar even when your client has confessed? a. Not addressed by ethics rules S. Statements by lawyers during jury trials 1. Rule 3.7 Lawyer as Witness a. A lawyer shall not act as advocate at a trial in which the lawyer is likely to be a necessary witness unless: i. the testimony relates to an uncontested issue ii. the testimony relates to the nature and value of legal services rendered in the case iii. disqualification of the lawyer would work substantial hardship on the client

52

b. A lawyer may act as advocate in a trial in which another lawyer in the lawyers firm is likely to be called as a witness unless precluded from doing so by Rule 1.7 or Rule 1.9 T. Rule 3.4(e) Fairness to Opposing Party and Counsel 1. A lawyer shall not: . . . in trial, allude to any matter that the lawyer does not reasonably believe is relevant or that will not be supported by admissible evidence, assert personal knowledge of facts in issue except when testifying as a witness, or state a personal opinion as to the justness of a cause, the credibility of a witness, the culpability of a civil litigant or the guilt or innocence of an accused U. Lawyers Duties in Nonadjudicative Proceedings 1. Rule 3.9 Advocate in Nonadjudicative Proceedings a. A lawyer representing a client before a legislative body or administrative agency in a nonadjudicative proceeding shall disclose that the appearance is in a representative capacity and shall conform to the provisions of Rules 3.3(a) through (c), 3.4(a) through (c) and 3.5(a) and (c). 2. See chart on page 549. V. Communications with Lawyers and Third Parties 1. Rule 4.1(a) Truthfulness in Statements to Others a. In the course of representing a client a lawyer shall not knowingly make a false statement of material fact or law to a third person 2. Problem 8-11: Emergency Food Stamps a. Did Professor Simon violate Rules 4.1 and 8.4(c)? i. Yesknowingly misrepresented himself/his practice via his paralegal, as Theresa Taylor ii. Argue deception is just to receptionist, so this is not material; issue if identity is material iii. Just be careful, be cautious b. If so, did he do the right thing? i. Yes, probably (subjective) 3. If a lawyer knows that her client or any witness she presents is lying to a tribunal or in a pretrial deposition, Rule 3.3(b) requires the lawyer to take remedial measures, which could include informing the tribunal if the client refuses to correct the record herself 4. In re Gatti a. By misrepresenting his identity and purpose and making other false statements when he called Becker and Adams with the intention of deceiving them, the accused violated his duty to the public to maintain personal integrity W. Restrictions on contact with represented partiesRule 4.2 1. Rule 4.2: Communication with Person Represented by Counsel a. In representing a client, a lawyer shall not communicate about the subject of the representation with a person the lawyer knows to be represented by another lawyer in the matter, unless the lawyer

53

has the a court

consent of the other lawyer or is authorized to do so by law or order i. Do not talk to anyone else represented, in that matter, about that matter b. Rule 4.2 applies whether the lawyer or the represented initiates the conversation i. Client cannot waive Rule 4.2 c. A lawyer may not attempt to violate any rule through the acts of another (no paralegal contact, no forced client contact with other

client) 2. Messing a. Messing, on behalf of Stanford, talked to two lieutenants, to other officers, and the dispatcher b. Court varied over who you should be able to talk to, and why c. You are interviewing them to find some sort of knowledge that will impute liability, some sort of act or omission d. Supreme Court does not interpret Rule 4.2 this broadly 3. An attorney may not speak ex parte to three categories of employees: (1) Supervises, directs or regularly consults with the organizations lawyer concerning the matter; (2) Has authority to obligate the organization with respect to the matter X. Restrictions on contact with unrepresented personsRule 4.3 1. Rule 4.3: Dealing with Unrepresented Person a. In dealing on behalf of a client with a person who is not represented by counsel, a lawyer shall not state or imply that the lawyer is disinterested. When the lawyer knows or reasonably should know that the unrepresented person misunderstands the lawyers role in the matter, the lawyer shall make reasonable efforts to correct the misunderstanding. b. The lawyer shall not give legal advice to an unrepresented person, other than the advice to secure counsel, if the lawyer knows or reasonably should know that the interests of such a person are or have a reasonable possibility of being in conflict with the interests of the client 2. Rule 4.3 also applies to a lawyer for a corporation and employees of the corporation 3. Can a lawyer encourage an unrepresented witness not to talk to her opposing counsel? a. Yes. Rule 4.3 prohibits a lawyer dealing on behalf of a client with a third person who has no lawyer from giving advice to that person if the interests of the witness may conflict with those of the client. If there is no such conflict, the lawyer may give this advice 4. Lawyers should not make secret recordings in violation of state law and secretly recording ones own client is at the least, inadvisable 5. Problem 8-12: The Prosecutors Masquerade

54

a. Cannot get more information from Flood; must let Flood know your role in the situation; cannot create a misunderstanding about your role to unrepresented persons b. If this were not an attorney, but a police officer, would we say this is okay? 6. Problem 8-13: The Complaining Witness a. Yes b. Fox should tell DiBello that she is not required to talk to Fox c. Yesbut DiBello may hear otherwise d. Yes e. Yes f. No, cannot say needdo not have to remind DiBello she may want outside counsel to review it Y. Conduct prejudicial to the administration 1. Rule 8.4(d) provides that it is professional misconduct for a lawyer to engage in conduct that is prejudicial to the administration of justice XXI. Exam A. Open rulesprint out Model Rules or WI rule supplement (Supreme Court) B. Fact patternsapply facts to the law/rules C. Graded on our ability to make an argument D. Few questions in which there is a right or wrong answer E. If you were going to do this, how would you use the rules/ethics to defend your behavior; if you were not going to do this, what rules/language would prevent this behavior? F. More than citing the appropriate rule. Talk about the impact if you choose a particular course of action; talk about ramifications of that choice, plusses and minuses of that choice G. A broader question that will ask some policiesbig picture question H. Do not retype or quote rules! I. Misrepresentation/Disclosure, Confidentiality, Conflicts of Interest, how do we weigh this rule against this rule? XXII. Legal Profession A. Problem 9-1: The Reforming Partner 1. Hire more associates; fewer hours 2. Give associates options for more pay for more hours B. Problem 9-2: The Job Interview 1. The interviewer asks Are you planning to start a family in the next few years? What will you say? a. Liesay right now, its not in the cards, focused on careers b. Ashley theoryif they lied first, you can lie back c. Choices i. Evade ii. Lie iii. Say it is an inappropriate question for an interview iv. Be truthful

55

v. Ask how the firm handles these situations/why the firm wants to know vi. Walk out XXIII. The Provision of Legal Services A. Problem 10-1: Special Education 1. Intent of Congress-this is an individual with special knowledge legal argument 2. Policy reasons to permit lay advocates here save parents money 3. Court said lay advocates here were acting as lawyers 4. Policy reason for not having this: some model ethical behavior expected; there are cases for malpractice, fraud, and there is some recourse; no recourse if a lay advocate B. Advertising 1. Problem 10-2: Do You Need a Lawyer? a. Solicitation within 30 days of an accident is banned C. Interstate law practice-page 649 D. Multidisciplinary practice 1. Offer combination of services (legal services, financial planning, tax) 2. Could ruin solo practitioners 3. Conflict and confidentiality rules would not apply to other professions

56

Você também pode gostar